Insight SUB CSP21T3S HIS MOD

You might also like

Download as pdf or txt
Download as pdf or txt
You are on page 1of 80

Total Marks : 200

Test-3 (Subject)
( INSTA Prelims Test Series 2021 )

1. Consider the following statements regarding Portuguese in India


1. The first governor of the Portuguese in India was Pedro Alvarez Cabral.
2. Pedro Alvarez Cabral negotiated and established a factory at Calicut.
3. Albuquerque captured Goa from the ruler of Bijapur and later became the capital of the
Portuguese settlements in India.

Which of the statements given above is/are correct?


A. 1 only
B. 1 and 3 only
C. 2 and 3 only
D. 1, 2 and 3

Correct Answer : C

Answer Justification :

A voyage was undertaken by Pedro Alvarez Cabral to trade for spices; he negotiated
and established a factory at Calicut, where he arrived in September 1500. There was an
incident of conflict when the Portuguese factory at Calicut was attacked by the locals,
resulting in the death of several Portuguese. In retaliation, Cabral seized a number of Arab
merchant ships anchored in the harbour, and killed hundreds of their crew besides
confiscating their cargo and burning the ships. Calicut was bombarded by Cabral. Later,
Cabral succeeded in making advantageous treaties with the local rulers of Cochin and
Cannanore.

The first governor of the Portuguese in India was Francis de Almeida. Hence,
statement 1 is incorrect. Later in 1509 Albuquerque was made the governor of the
Portuguese territories in India. In 1510, he captured Goa from the ruler of Bijapur.
Thereafter, Goa became the capital of the Portuguese settlements in India. Albuquerque
captured Malacca and Ceylon. He also built a fort at Calicut. He encouraged his countrymen to
marry Indian women. Albuquerque died in 1515 leaving the Portuguese as the strongest naval
power in India. Almeida’s vision was to make the Portuguese the master of the Indian Ocean.
His policy was known as the Blue Water Policy (cartaze system).

2. Consider the following events


1. Annexation of Burma by British
2. The Second Afghan War by British
3. Invasion of Tibet under Curzon

Which of the following is the correct chronological sequence of the above events?
A. 1-2-3
B. 3-2-1
C. 1-3-2
D. 2-1-3

www.insightsactivelearn.com 1
Total Marks : 200
Test-3 (Subject)
( INSTA Prelims Test Series 2021 )

Correct Answer : D

Answer Justification :

After 1878, the British undertook a number of expansionist expeditions which were opposed by
the nationalists. These expeditions included—

● the Second Afghan War (1878-80);

● the dispatch of troops by England in 1882, to suppress the nationalist uprising by Col. Arabi
inEgypt;

● annexation of Burma in 1885;

● invasion of Tibet under Curzon in 1903; and

● a number of annexations during the 1890s in the north-west to stop the Russian advance.
The nationalists supported the tribal resistance to these adventures by the British

3. Consider the following statements regarding the Brahmo Samaj:


1. It was founded by Raja Rammohan Roy
2. It opposed idol worship.
3. It supported widow remarriage.

Which of the statements given above is/are correct?


A. 1 and 2 only
B. 2 and 3 only
C. 1 and 3 only
D. 1, 2 and 3

Correct Answer : D

Answer Justification :

All the statements given above are correct.

Raja Rammohan Roy founded the Brahmo Sabha in 1828 which later came to be
known as Brahmo Samaj. It was open to all people regardless of their colour, convictions,
caste, nationality, and religion. It emphasised human dignity, opposed idol worship and
condemned social evils like sati pratha. It was not meant to be a separate religious sect but
only a place where all those who believed in one true God could meet and pray. No images
were allowed and no sacrifices and offerings permitted.

It denounced polytheism, idol worship, and the faith in divine avatars (incarnations). It
condemned the caste system, dogmas and superstitions. It wanted the abolition of child

www.insightsactivelearn.com 2
Total Marks : 200
Test-3 (Subject)
( INSTA Prelims Test Series 2021 )

marriage, purdah system and the practice of sati. It supported widow remarriage.

Debendra Nath Tagore founder member of Brahmo Samaj, succeeded Raja Rammohan Roy as
the leader of the Brahmo Samaj. He put new life in the Samaj and propagated Raja Rammohan
Roy’s ideas. Keshub Chandra Sen took over the leadership of the Samaj from Tagore.

4. Consider the following statements regarding Dutch East India Company


1. The Dutch East India Company was established in 1602.
2. The Dutch founded their first factory in Masulipatnam (in Andhra) in 1605.

Which of the statements given above is/are correct?


A. 1 only
B. 2 only
C. Both 1 and 2
D. Neither 1 nor 2

Correct Answer : C

Answer Justification :

The Dutch East India Company was established in 1602. Commercial enterprise led the
Dutch to undertake voyages to the East. Cornelis de Houtman was the first Dutchman to reach
Sumatra and Bantam in 1596. In 1602, the States- General of the Netherlands amalgamated
many trading companies into the East India Company of the Netherlands.

After their arrival in India, the Dutch founded their first factory in Masulipatnam (in
Andhra) in 1605. They went on to establish trading centres in different parts of India and
thus became a threat to the Portuguese. They captured Nagapatam near Madras (Chennai)
from the Portuguese and made it their main stronghold in South India.

5. Consider the following statements regarding the Thalassemia disease:


1. It is an inherited blood disorder.
2. It is caused when the body doesn’t make enough of hemoglobin.

Which of the statements given above is/are correct?


A. 1 only
B. 2 only
C. Both 1 and 2
D. Neither 1 nor 2

Correct Answer : C

www.insightsactivelearn.com 3
Total Marks : 200
Test-3 (Subject)
( INSTA Prelims Test Series 2021 )

Answer Justification :

Both the statements given above are correct.

Thalassemia is an inherited (i.e., passed from parents to children through genes)


blood disorder caused when the body doesn’t make enough of a protein called
hemoglobin, an important part of red blood cells.

When there isn’t enough hemoglobin, the body’s red blood cells don’t function properly and
they last for shorter periods of time, so there are fewer healthy red blood cells traveling in the
bloodstream.

When there are not enough healthy red blood cells, there is also not enough oxygen
delivered to all the other cells of the body, which may cause a person to feel tired,
weak or short of breath.

People with thalassemia might have mild or severe anemia. Severe anemia can damage
organs and lead to death.

https://medicaldialogues.in/state-news/delhi/dr-harsh-vardhan-inaugurates-thalassemia-screeni
ng-and-counselling-centre-at-indian-red-cross-68718

https://pib.gov.in/PressReleasePage.aspx?PRID=1646731

6. With reference to Nationalist’s response to British’s foreign policy, consider the following statements
1. Nationalists condemned the imperialist efforts to divide China by foreign forces.
2. They expressed the solidarity with other colonies fighting for freedom such as Russia and
Ireland.
3. Congress supported for Burma’s freedom

Which of the statements given above is/are correct?


A. 2 only
B. 1, 2 and 3
C. 2 and 3 only
D. 3 only

Correct Answer : B

Answer Justification :

All the statements given above are correct.

In place of an aggressive imperialism of British forces, the nationalists advocated a policy of


peace. C. Sankaran Nair, the Congress president in 1897, said, “Our true policy is a peaceful
policy.” So, the emerging themes during 1880-1914 were—

www.insightsactivelearn.com 4
Total Marks : 200
Test-3 (Subject)
( INSTA Prelims Test Series 2021 )

1. Solidarity with other colonies fighting for freedom, such as Russia, Ireland, Egypt,
Turkey, Ethiopia, Sudan, Burma and Afghanistan;

2. pan-Asian feeling reflected in—

● Condemnation of annexation of Burma in 1885,

● Inspiration from Japan as an example of industrial development,

● Condemnation of the participation of Japan in the international suppression of the I-Ho-Tuan


uprising (1895),

● Condemnation of the imperialist efforts to divide China

● Congress support for Burma’s freedom.

7. Consider the following statements regarding arrival of British in India.


1. The English East India Company was established in 1600.
2. Captain Hawkins arrived at the royal court of Jahangir to seek permission to establish English
trading centre at Calcutta.
3. The ‘Golden Farman’ issued by the Sultan of Golconda is regarded the Magna Carta of the
Company.

Which of the statements given above is/are correct?


A. 1 only
B. 1 and 3 only
C. 2 and 3 only
D. 1, 2 and 3

Correct Answer : A

Answer Justification :

The English East India Company was established in 1600 and the Charter was issued by
Queen Elizabeth of England. Captain Hawkins arrived at the royal court of Jahangir in
1609 to seek permission to establish English trading centre at Surat. Hence,
statement 2 is incorrect. But it was refused by the Mughal Emperor due to Portuguese
pressure. Later in 1612, Jahangir issued a farman (permission letter) to the English and they
established a trading factory at Surat in 1613.

Farrukhsiyar’s Farmans:

In 1715, an English mission led by John Surman to the court of the Mughal emperor
Farrukhsiyar secured three famous farmans, giving the Company many valuable privileges in

www.insightsactivelearn.com 5
Total Marks : 200
Test-3 (Subject)
( INSTA Prelims Test Series 2021 )

Bengal, Gujarat and Hyderabad. The farmans thus obtained were regarded as the Magna
Carta of the Company. Hence, statement 3 is incorrect.

The English company’s position was improved by the ‘Golden Farman’ issued to them by the
Sultan of Golconda in 1632. On a payment of 500 pagodas a year, they earned the privilege of
trading freely in the ports of Golconda. A member of the Masulipatnam council, the British
merchant Francis Day in 1639 received from the ruler of Chandragiri permission to build a
fortified factory at Madras which later became the Fort St. George and replaced Masulipatnam
as the headquarters of the English settlements in south India.

8. Consider the following statements regarding Pandita Ramabai:


1. She set up the Brahmo Mahila Samaj in Poona.
2. She brought out a book in Marathi, Stree Dharma Niti.

Which of the statements given above is/are correct?


A. 1 only
B. 2 only
C. Both 1 and 2
D. Neither 1 nor 2

Correct Answer : B

Answer Justification :

Pandita Ramabai (1858-1922) was one of the greatest women of modern India. Exceptionally
learned, Ramabai, an outspoken champion of women's rights and social reform.

She set up the Arya Mahila Samaj in 1882 in Poona to mobilise women and aroused
instant hostility. Hence, statement 1 is incorrect.

She brought out a book in Marathi, Stree Dharma Niti [Morals for Women] with the
objective of counselling the helpless and ignorant women. Hence, statement 2 is
correct.

Ramabai set up a home for high-caste Hindu widows (Mukti Sadan) and made an appeal
to the Hunter Commission to provide training facilities to women to become teachers and
doctors enabling them to serve other women.

9. With reference to the Nationalist response to World Wars, consider the following statements
1. The nationalists didn’t support the British Indian Government in World War I
2. Congress condemned the dispatch of Indian Army to suppress the Chinese nationalist army
under SunYat-Sen.
3. The Japanese attack on China was condemned by the nationalists.

www.insightsactivelearn.com 6
Total Marks : 200
Test-3 (Subject)
( INSTA Prelims Test Series 2021 )

Which of the statements given above is/are correct?


A. 2 only
B. 2 and 3 only
C. 3 only
D. 1, 2 and 3

Correct Answer : B

Answer Justification :

World War I

The nationalists supported the British Indian Government in the belief that Britain
would apply the same principles of democracy for which they were supposed to be
fighting. Hence Statement 1 is incorrect.

After the conclusion of the War, the Congress insisted on being represented at the Peace
Conference. In 1920, the Congress urged the people not to join the Army to fight in the West.

In 1925, the Congress condemned the dispatch of Indian Army to suppress the
Chinese nationalist army under SunYat-Sen. Hence Statement 2 is correct.

In 1939, the Japanese attack on China was condemned by the nationalists. The
Congress also sent a medical mission under Dr Atal to China. Hence Statement 3 is
correct.

10. Which of the following group of countries is/are members of the Quadrilateral Security Dialogue (QUAD)
group?

A. Qatar, UAE, Algeria and Djibouti


B. India, USA, Japan and Australia
C. China, Russia, Uzbekistan and Pakistan
D. India. China, Russia and South Korea

Correct Answer : B

Answer Justification :

The Quadrilateral Security Dialogue (QSD, also known as the Quad) is an informal
strategic forum between the United States, Japan, Australia and India that is
maintained by semi-regular summits, information exchanges and military drills between
member countries. Hence, option (b) is correct.

The forum was initiated as a dialogue in 2007 by Prime Minister Shinzo Abe of Japan,

www.insightsactivelearn.com 7
Total Marks : 200
Test-3 (Subject)
( INSTA Prelims Test Series 2021 )

with the support of Vice President Dick Cheney of the US, Prime Minister John
Howard of Australia and Prime Minister Manmohan Singh of India. The dialogue was
paralleled by joint military exercises of an unprecedented scale, titled Exercise
Malabar. The diplomatic and military arrangement was widely viewed as a response to
increased Chinese economic and military power, and the Chinese government responded to
the Quad by issuing formal diplomatic protests to its members.

11. Consider the following statements regarding French settlements in India


1. The first French factory in India was established at Surat.
2. Francois Martin founded Pondicherry in 1673 and was the first governor of Pondicherry.

Which of the statements given above is/are correct?


A. 1 only
B. 2 only
C. Both 1 and 2
D. Neither 1 nor 2

Correct Answer : C

Answer Justification :

The French were the last Europeans to come to India with the purpose of trade.
During the reign of Louis XIV, the king’s famous minister Colbert laid the foundation of the
Compagnie des Indes Orientales (French East India Company) in 1664, in which the king also
took a deep interest. The Compagnie des Indes Orientales was granted a 50-year monopoly on
French trade in the Indian and Pacific Oceans.

The French East India Company was formed in 1664 by Colbert, a Minister under Louis XIV.
The first French factory in India was established at Surat by Francis Caron. Later,
Maracara set up a factory at Masulipattinam. Francois Martin founded Pondicherry in
1673. Other French factories in India were Chandranagore, Mahe and Karaikal.
Francois Martin was the first governor of Pondicherry, the headquarters of the
French possessions in India.

12. Which of the following criteria’s a country must meet to become a part of Non-Aligned Movement?
1. A country should follow an independent policy based on peaceful co-existence and non-
alignment.
2. It should have consistently supported national freedom movements in other countries.

Select the correct answer using the code given below


A. 1 only
B. 2 only
C. Both 1 and 2

www.insightsactivelearn.com 8
Total Marks : 200
Test-3 (Subject)
( INSTA Prelims Test Series 2021 )

D. Neither 1 nor 2

Correct Answer : C

Answer Justification :

Both the statements given above are correct.

Non-alignment is the characteristic feature of India’s foreign policy. India was one of the
founder-members of NAM.

In the Cold War era, India refused to favour any super power and remained non-aligned. Non-
alignment, however, is not to be confused with neutrality. A neutral state remains inactive or
passive during hostilities between two blocs.

Neutrality is maintained basically in times of war, whereas non-alignment has relevance both
in times of war and peace.

Neutrality is equivalent to passivity, a neutral country has no opinions (positive or negative) on


issues at all.

The Preparatory Committee of the first non-aligned conference laid


down the following five criteria of non-alignment:

(i) A country should follow an independent policy based on peaceful co-existence and
non-alignment.
(ii) It should have consistently supported national freedom movements in other
countries.
(iii) It should not be a member of multi-lateral military alliances concluded in the context of
super-power conflicts.
(iv) If it has conceded military bases, these concessions should not have been made in the
context of super-power conflicts.
(v) If it is a member of a bilateral or regional defence arrangements, this should not be in the
context of super-power politics

13. Consider the following statements regarding the political organizations during 19th century in India:
1. The Bangabhasha Prakasika Sabha was formed in 1836 by associates of Rammohun Roy.
2. Indian league was started in Calcutta by Ishwar Chandra Vidyasagar.
3. Bengal British India Society was set up in 1843 by Sisir Kumar Ghosh in London.

Which of the statements given above is/are correct?


A. 1 and 2 only
B. 1 only
C. 3 only

www.insightsactivelearn.com 9
Total Marks : 200
Test-3 (Subject)
( INSTA Prelims Test Series 2021 )

D. 1, 2 and 3

Correct Answer : B

Answer Justification :

The Bangabhasha Prakasika Sabha was formed in 1836 by associates of Rammohun Roy.
The main aim of the organization was to promote Bengali education by means of
polemics and build up public opinion. Hence, statement 1 is correct.

On 25 September 1875 the Indian League was founded under the leadership of Sisir
Kumar Ghosh. The membership fee of the League was RS.5/-, but for workers, peasants and
people of the villages, the rate of subscription was lowered to One rupee. Sambhu Chandra
Mukerjee became the President, Kalinath Das, the Secretary, and Sisir Kumar Ghosh,
the assistant secretary, of the League. Hence, statement 2 is incorrect.

Bengal British India Society founded in Calcutta in 1843 by William Adam, a friend of
Raja Rammohan Roy in England. The Bengal British India Society was an organisation
dominated by a section of the Bengal intellectuals, particularly by the young Bengal group.
Hence, statement 3 is incorrect.

14. Which of the following are the causes for the English success over French?
1. The English company was a private enterprise whereas the French company was controlled
and regulated by the French government.
2. The English navy was superior to the French navy.
3. The English held three important places, namely, Calcutta, Bombay and Madras whereas the
French had only Pondicherry.
4. The French subordinated their commercial interest to territorial ambition, which made the
French company short of funds.

Select the correct answer using the code given below:


A. 1, 2 and 3 only
B. 2, 3 and 4 only
C. 1, 2 and 4 only
D. 1, 2, 3 and 4

Correct Answer : D

Answer Justification :

All the above statements are correct.

Causes for the English Success and the French Failure

www.insightsactivelearn.com 10
Total Marks : 200
Test-3 (Subject)
( INSTA Prelims Test Series 2021 )

The English company was a private enterprise—this created a sense of enthusiasm


and self-confidence among the people. With less governmental control over it, this
company could take instant decisions when needed without waiting for the approval of
the government. The French company, on the other hand, was a State concern. It
was controlled and regulated by the French government and was hemmed in by
government policies and delays in decision-making.
The English navy was superior to the French navy; it helped to cut off the vital sea
link between the French possessions in India and France.
The English held three important places, namely, Calcutta, Bombay and Madras
whereas the French had only Pondicherry.
The French subordinated their commercial interest to territorial ambition, which
made the French company short of funds.
In spite of their imperialistic motives, the British never neglected their commercial
interests. So they always had the funds and the consequent sound financial condition to
help them significantly in the wars against their rivals.
A major factor in the success of the English in India was the superiority of the
commanders in the British camp.
In comparison to the long list of leaders on the English side—Sir Eyre Coote, Major
Stringer Lawrence, Robert Clive and many others—there was only Dupleix on the French
side.

15. Consider the following statements regarding the Pradhan Mantri Kisan Samman Nidhi (PM-KISAN):
1. It is a Centrally Sponsored scheme.
2. The entire responsibility of identification of beneficiary farmer families rests with the State /
UT Governments.
3. The fund is directly transferred to the bank accounts of the beneficiaries.

Which of the statements given above is/are correct?


A. 1 and 2 only
B. 2 and 3 only
C. 1 and 3 only
D. 1, 2 and 3

Correct Answer : B

Answer Justification :

Pradhan Mantri Kisan Samman Nidhi (PM-KISAN):

Pradhan Mantri Kisan Samman Nidhi (PM-KISAN) is a Central Sector scheme


with 100% funding from Government of India.
The Scheme is effective from 1.12.2018. Hence, statement 1 is incorrect.
Under the Scheme an income support of Rs.6000/- per year is provided to all
farmer families across the country in three equal installments of Rs.2000/- each,
every four months.

www.insightsactivelearn.com 11
Total Marks : 200
Test-3 (Subject)
( INSTA Prelims Test Series 2021 )

Definition of family for the Scheme is husband, wife and minor children.
The entire responsibility of identification of beneficiary farmer families rests
with the State / UT Governments. Hence, statement 2 is correct.
The fund is directly transferred to the bank accounts of the beneficiaries.
Hence, statement 3 is correct.
Farmers covered under the Exclusion Criteria of the Operational Guidelines are not
eligible for the benefit of the Scheme.
For enrollment, the farmer is required to approach the local patwari / revenue officer /
Nodal Officer (PM-Kisan) nominated by the State Government.
The Common Service Centres (CSCs) have also been authorized to do registration of the
farmers for the Scheme upon payment of fees.
Farmers can also do their self-registration through the Farmers Corner in the portal.
Farmers can also edit their names in PM-Kisan database as per their Aadhaar database /
card through the Farmers Corner in the portal.
Farmers can also know the status of their payment through the Farmers Corner in the
portal.

https://indianexpress.com/article/opinion/columns/pm-kisan-farmer-scheme-lockdown-6536208

16. Which of the following is/are the principles of Panchsheel?


1. Mutual non-aggression
2. Mutual co-operation in the matter of internal issues.
3. Mutual support in case of foreign aggression.

Select the correct answer using the code given below


A. 1, 2 and 3
B. 1 only
C. 1 and 3 only
D. None

Correct Answer : B

Answer Justification :

It was on April 29, 1954, that Panchsheel, or the Five Principles of Peaceful Co-existence, was
first formally enunciated in the Agreement on Trade and Intercourse between the Tibet region
of China and India.

It was stated in the preamble to this agreement that the two governments had resolved to
enter into the agreement on the basis of five principles, namely,

(i) Mutual respect for each other’s territorial integrity and sovereignty

(ii) Mutual non-aggression

www.insightsactivelearn.com 12
Total Marks : 200
Test-3 (Subject)
( INSTA Prelims Test Series 2021 )

(iii) Mutual non-interference

(iv) Equality and mutual benefit

(v) Peaceful co-existence. Hence Statement 2 and 3 are incorrect.

17. Consider the following statements regarding Danes rule in India:


1. The Danes were the last Europeans to come to India with the purpose of trade.
2. The Danes are better known for their missionary activities than for commerce.

Which of the statements given above is/are correct?


A. 1 only
B. 2 only
C. Both 1 and 2
D. Neither 1 nor 2

Correct Answer : B

Answer Justification :

The Danish East India Company was established in 1616 and, in 1620, they founded a
factory at Tranquebar near Tanjore, on the eastern coast of India. Their principal settlement
was at Serampore near Calcutta. The Danish factories, which were not important at any time,
were sold to the British government in 1845. The Danes are better known for their
missionary activities than for commerce.

The French were the last Europeans to come to India with the purpose of trade.
Hence, statement 1 is incorrect.

18. Which of the following organization started the ‘Shuddhi movement’?

A. Brahmo Samaj
B. Satya Shodak Samaj
C. Arya Samaj
D. Indian Social Conference

Correct Answer : C

Answer Justification :

www.insightsactivelearn.com 13
Total Marks : 200
Test-3 (Subject)
( INSTA Prelims Test Series 2021 )

Shuddhi movement’ was started by the Arya Samaj, and its founder Swami Dayanand
Saraswati and his followers such as Swami Shraddhanand.

The objective of the movement was to abolish the practise of untouchability by


converting outcasts from other religions to Hinduism and integrating them into the
mainstream community by elevating their position. The movement also aimed to reduce
the conversions of Hindus to Islam and Christianity,

Hence, option (c) is correct.

19. “The misery hardly finds a parallel in the history of commerce; the bones of cotton weavers are bleaching the
plains of north India.”
Which of the following personality expressed the above view in the context of economic policies of the
British?

A. D.H. Buchanan
B. William Bentinck
C. Shashi Tharoor
D. Lord Macaulay

Correct Answer : B

Answer Justification :

With the end of its monopoly in 1813, the import of mill-made goods from Britain
devastated the once-mighty Indian textile industry. "The misery hardly finds parallel
in the history of commerce," said the governor general, William Bentinck. "The bones
of the cotton weavers are bleaching the plains of India."

20. Alliance 8.7, sometimes seen in news, aims to eradicate:


1. Forced Labour
2. Human Trafficking
3. Child Labour

Select the correct answer using the code given below:


A. 1 and 2 only
B. 2 and 3 only
C. 1 and 3 only
D. 1, 2 and 3

Correct Answer : D

www.insightsactivelearn.com 14
Total Marks : 200
Test-3 (Subject)
( INSTA Prelims Test Series 2021 )

Answer Justification :

All the statements given above are correct.

Alliance 8.7 is a global partnership fostering multi-stakeholder collaboration to


support governments in achieving target 8.7 of Agenda 2030. It promotes 1)
accelerated action to eradicate forced labour, modern slavery, human trafficking and
child labour; 2) research, data collection and knowledge sharing on prevalence and “what
works”; and 3) driving innovation and leveraging resources. The Alliance works globally
through four thematic Action Groups, a Communication Group and supports the national
efforts of an initial group of 15 pathfinder countries that have committed to accelerate action,
organize national multi-stakeholder consultations and set up respective time-bound action
plans with measurable targets.

Alliance 8.7 builds on multi-stakeholder commitment and collaborative responsibility for


resource mobilization efforts. As described above, implementation is principally based on
coordinated national activities in pathfinder countries and global support through Action
Groups. While Pathfinder countries are ultimately responsible for implementing the national
action plans they develop in coordination with other Alliance 8.7 Partners and stakeholders,
four thematic Action Groups support these efforts in different areas of implementation: 1)
Humanitarian Settings and Conflict; 2) Supply Chains; 3) Migration and 4) Rule of Law and
Governance.

https://www.thehindu.com/opinion/editorial/historic-ratification-the-hindu-editorial-on-universa
l-ratification-of-a-labour-standard/article32347515.ece

https://sustainabledevelopment.un.org/partnership/?p=32967

21. Consider the following statements regarding British settlements in India


1. Francis Day established the city of Madras and constructed the Fort St. George.
2. The city of Calcutta was fortified by Job Charnock and named it Fort William.

Which of the statements given above is/are correct?


A. 1 only
B. 2 only
C. Both 1 and 2
D. Neither 1 nor 2

Correct Answer : C

Answer Justification :

In 1639, Francis Day established the city of Madras and constructed the Fort St.
George. On the west coast, the Company obtained Bombay on lease from their King, Charles
II for a rent of 10 pounds per annum in 1668. By the year 1690, Job Charnock, the agent of

www.insightsactivelearn.com 15
Total Marks : 200
Test-3 (Subject)
( INSTA Prelims Test Series 2021 )

the East India Company

purchased three villages namely, Sutanuti, Govindpur and Kalikatta, which, in course
of time, grew into the city of Calcutta. It was fortified by Job Charnock, who named it
Fort William after the English King, William III. The factories and trading centres which
the English established all along the sea-coast of India were grouped under three presidencies
namely Bombay, Madras and Calcutta.

22. With reference to the Economic exploitation of British, consider the following statements
1. The drain theory was put forward by Romesh Chandra Dutt.
2. The major components of drain were salaries and pensions of civil and military officials,
interests on loans taken by the Indian Government from abroad.
3. The drain of wealth checked and retarded capital formation in India.

Which of the statements given above is/are correct?


A. 1 and 2 only
B. 2 and 3 only
C. 3 only
D. 1, 2 and 3

Correct Answer : B

Answer Justification :

The term ‘economic drain’ refers to a portion of national product of India which was not
available for consumption of its peoples, but was being drained away to Britain for political
reasons and India was not getting adequate economic or material returns for it.

The drain theory was put forward by Dadabhai Naoroji in his book Poverty and
UnBritish Rule in India. Hence Statement 1 is incorrect.

The major components of this drain were salaries and pensions of civil and military
officials, interests on loans taken by the Indian Government from abroad, profits on
foreign investment in India, stores purchased in Britain for civil and military departments,
payments to be made for shipping, banking and insurance services which stunted the growth
of Indian enterprise in these services. Hence Statement 2 is correct.

The drain of wealth checked and retarded capital formation in India while the same portion of
wealth accelerated the growth of British economy. Hence Statement 3 is correct.

23. Consider the following statements regarding Savitribai Phule:


1. Phule along with her husband started one of the first Indian girls' school in Poona.
2. She set up a home, Balhatya Pratibandhak Griha, to prevent the killing of widows.

www.insightsactivelearn.com 16
Total Marks : 200
Test-3 (Subject)
( INSTA Prelims Test Series 2021 )

Which of the statements given above is/are correct?


A. 1 only
B. 2 only
C. Both 1 and 2
D. Neither 1 nor 2

Correct Answer : C

Answer Justification :

Both the statements given above are correct.

Savitribai Phule was a crusader for women empowerment. She contributed immensely in the
field of education of women and women empowerment.

Phule and her husband started one of the first Indian girls' school in Pune, at Bhide
wada in 1848. She is remembered as India’s first female teacher who worked for the
upliftment of women in terms of education and literacy.

Along with her husband, she also worked towards preventing female infanticide and
set up a home, Balhatya Pratibandhak Griha, to prevent the killing of widows. She also
campaigned against child marriage and sati pratha, which undermined the existence of
women.

She gave stipends to children for attending school in order to control the school dropout rate.

To honour her contribution, the University of Pune was renamed as Savitribai Phule
Pune University.

24. Consider the following statements regarding administration before the enactment of The Regulating Act of
1773
1. In India, each of the three presidencies was independent and responsible only to the Home
Government.
2. The Home government in England consisted of the Court of Directors and the Court of
Proprietors.
3. The Court of Directors were elected annually and practically managed the affairs of the
Company.

Which of the statements given above is/are correct?


A. 1 and 2 only
B. 1 and 3 only
C. 2 and 3 only
D. 1, 2 and 3

www.insightsactivelearn.com 17
Total Marks : 200
Test-3 (Subject)
( INSTA Prelims Test Series 2021 )

Correct Answer : D

Answer Justification :

The Regulating Act of 1773 opened a new chapter in the constitutional history of the Company.
Previously, the Home government in England consisted of the Court of Directors and
the Court of Proprietors. The Court of Directors were elected annually and practically
managed the affairs of the Company. In India, each of the three presidencies was
independent and responsible only to the Home Government. The government of the
presidency was conducted by a Governor and a Council.

The following conditions invited the Parliamentary intervention in the Company’s affairs. The
English East India Company became a territorial power when it acquired a wide dominion in
India and also the Diwani rights. Its early administration was not only corrupt but notorious.
When the Company was in financial trouble, its servants were affluent. The disastrous famine
which broke out in Bengal in 1770 affected the agriculturists. As a result, the revenue
collection was poor. In short, the Company was on the brink of bankruptcy. In 1773, the
Company approached the British government for an immediate loan. It was under these
circumstances that the Parliament of England resolved to regulate the affairs of the Company.
Lord North, the Prime Minister of England, appointed a select committee to inquire into the
affairs of the Company. The report submitted by the Committee paved the way for the
enactment of the Regulating Act.

25. Agatti Island, recently seen in news, is located in:

A. Andaman and Nicobar group of Islands


B. Lakshadweep
C. Gulf of Khambhat
D. Gulf of Mannar

Correct Answer : B

Answer Justification :

Agatti Island is a 7.6 km long island, situated on a coral atoll called Agatti atoll in the
Union Territory of Lakshadweep, India.

Recently, the National Green Tribunal granted an interim stay on felling of coconut
trees on Agatti Island for the purpose of forming a Beach Road.

www.insightsactivelearn.com 18
Total Marks : 200
Test-3 (Subject)
( INSTA Prelims Test Series 2021 )

https://www.thehindu.com/news/national/other-states/ngt-stays-felling-of-coconut-trees-on-laks
hadweep-island/article32253771.ece

26. Which of the following personalities was/were the critics of economic policies of British?
1. Dadabhai Naoroji
2. Prabhat Kumar Roy
3. G. Subramaniya Iyer

Select the correct answer using the code given below


A. 2 only
B. 1 and 3 only
C. 3 only
D. 1, 2 and 3

Correct Answer : B

Answer Justification :

www.insightsactivelearn.com 19
Total Marks : 200
Test-3 (Subject)
( INSTA Prelims Test Series 2021 )

The foremost among these economic analysts was Dadabhai Naoroji, the ‘Grand Old Man of
India’, who after a brilliant analysis of the colonial economy put forward the theory of
economic drain in his booked titled - Poverty and UnBritish Rule in India.

Other economic analysts included Justice Mahadeo Govind Ranade, Romesh Chandra
Dutt (The Economic History of India), Gopal Krishna Gokhale, G. Subramaniya Iyer
and Prithwishchandra Ray. The essence of nineteenth century colonialism, they said, lay in
the transformation of India into a supplier of foodstuffs and raw-materials to the metropolis, a
market for metropolitan manufacturers and a field for investment of British capital.

Prabhat Kumar Roy was not an economic critique of British Policies.

27. Consider the following statements regarding the Regulating Act of 1773
1. The Act brought the affairs of the Company under the control of the Parliament.
2. It proved that the Parliament of England was concerned about the welfare of Indians.
3. The main defect of the Act was that the Governor-General was made powerful and created
arbitrary rule.

Which of the statements given above is/are correct?


A. 1 and 2 only
B. 1 and 3 only
C. 2 and 3 only
D. 1, 2 and 3

Correct Answer : A

Answer Justification :

The Regulating Act of 1773:

Provisions of the Act:

The Regulating Act reformed the Company’s Government at Home and in India. The important
provisions of the Act were:

(i) The term of office of the members of the Court of Directors was extended from one year to
four years. One-fourth of them were to retire every year and the retiring Directors were not
eligible for re-election.

(ii) The Governor of Bengal was styled the Governor-General of Fort William whose tenure of
office was for a period of five years.

(iii) A council of four members was appointed to assist the Governor-General. The government
was to be conducted in accordance with the decision of the majority. The Governor- General

www.insightsactivelearn.com 20
Total Marks : 200
Test-3 (Subject)
( INSTA Prelims Test Series 2021 )

had a casting vote in case of a tie.

(iv) The Governor-General in Council was made supreme over the other Presidencies in
matters of war and peace.

(v) Provision was made in the Act for the establishment of a Supreme Court at Calcutta
consisting of a Chief Justice and three junior judges. It was to be independent of the Governor-
General in Council. In 1774, the Supreme Court was established by a Royal Charter.

(vi) This Act prevented the servants of the Company including the Governor-General, members
of his council and the judges of the Supreme Court from receiving directly or indirectly any
gifts in kind or cash.

Merits and Demerits of the Act

The significance of the Regulating Act is that it brought the affairs of the Company
under the control of the Parliament. Besides, it proved that the Parliament of
England was concerned about the welfare of Indians. The greatest merit of this Act is
that it put an end to the arbitrary rule of the Company and provided a framework for all future
enactments relating to the governing of India.

The main defect of the Act was that the Governor-General was made powerless
because the council which was given supreme power often created deadlocks by over-
ruling his decision. However, many of these defects were rectified by the Pitt’s India Act of
1784. Hence, statement 3 is incorrect.

28. Consider the following statements regarding Ramabai Ranade:


1. She started 'Hindu Ladies Social and Literary Club' in Bombay.
2. She was one of the founding members of Seva Sadan.
3. She wrote ‘Stree Purush Tulana’ (or Comparison of Men and Women).

Which of the statements given above is/are correct?


A. 1 and 2 only
B. 2 and 3 only
C. 1 and 3 only
D. 1, 2 and 3

Correct Answer : A

Answer Justification :

Ramabai Ranade was an Indian social worker and one of the first womens' rights activists in
the 19th century.

Inspired by her husband, Ramabai started 'Hindu Ladies Social Club' in Bombay to

www.insightsactivelearn.com 21
Total Marks : 200
Test-3 (Subject)
( INSTA Prelims Test Series 2021 )

develop public speaking among women. Ramabai was also a founder and President of
'Seva Sadan Society' in Pune. Hence, both statement 1 and statement 2 are correct.

Ramabai devoted her life to the improvement of women's lives. Ramabai Ranade with her
husband and other colleagues established in 1886 the first girls' high school in Pune.

She established a branch of Arya Mahila Samaj in Bombay.

At the urging of Ramakrishna Gopal Bhandarkar and Mr. Bhajekar, Ramabai chaired the
first session of India Women Conference held in Bombay in 1904.

Stree Purush Tulana (or Comparison of Men and Women) was written by Tarabai
Shinde, as a protest against the double standards of a male dominated society. Hence,
statement 3 is incorrect.

29. Consider the following statements regarding the provisions of Pitt’s India Act, 1784
1. A Board of Control consisting of six members was created and they were appointed by the
Crown.
2. It increased the number of the members of the Governor-General’s Council from three to six.

Which of the statements given above is/are correct?


A. 1 only
B. 2 only
C. Both 1 and 2
D. Neither 1 nor 2

Correct Answer : A

Answer Justification :

Pitt’s India Act, 1784:

The Regulating Act proved to be an unsatisfactory document as it failed in its objective. In


January 1784, Pitt the Younger (who became Prime Minister of England after the General
Elections) introduced the India Bill in the British Parliament. Despite bitter debate in both the
Houses, the bill was passed after seven months and it received royal assent in August 1784.
This was the famous Pitt’s India Act of 1784.

Main Provisions:

(i) A Board of Control consisting of six members was created. They were appointed by
the Crown.

(ii) The Court of Directors was retained without any alteration in its composition.

(iii) The Act also introduced significant changes in the Indian administration. It reduced the

www.insightsactivelearn.com 22
Total Marks : 200
Test-3 (Subject)
( INSTA Prelims Test Series 2021 )

number of the members of the Governor-General’s Council from four to three


including the Commander-in-Chief. Hence, statement 2 is incorrect.

Pitt’s India Act constitutes a significant landmark with regard to the foreign policy of the
Company. A critical review of the Act reveals that it had introduced a kind of contradiction in
the functions of the Company. The Court of Directors controlled its commercial functions,
whereas the Board of Control maintained its political affairs. In fact, the Board represented
the King, and the Directors symbolized the Company.

30. Consider the following statements regarding the Monetary Policy Committee (MPC):
1. It was constituted under the Reserve Bank of India Act, 1934
2. The Union Finance Minister is the ex-officio chairperson of the committee.
3. The meetings of the Monetary Policy Committee shall be held at least two times a year.

Which of the statements given above is/are correct?


A. 1 only
B. 2 and 3 only
C. 1, 2 and 3
D. None

Correct Answer : A

Answer Justification :

The Reserve Bank of India Act, 1934 (RBI Act) has been amended by the Finance Act,
2016, to provide for a statutory and institutionalised framework for a Monetary Policy
Committee, for maintaining price stability, while keeping in mind the objective of
growth. Hence, statement 1 is correct.

The Monetary Policy Committee would be entrusted with the task of fixing the benchmark
policy rate (repo rate) required to contain inflation within the specified target level. A
Committee-based approach for determining the Monetary Policy will add lot of value and
transparency to monetary policy decisions. The meetings of the Monetary Policy
Committee shall be held at least 4 times a year and it shall publish its decisions after
each such meeting. Hence, statement 2 is incorrect.

As per the provisions of the RBI Act, out of the six Members of Monetary Policy
Committee, three Members will be from the RBI and the other three Members of MPC
will be appointed by the Central Government.

The Governor of the Reserve Bank of India (RBI) is the ex officio Chairperson of the
committee. Hence, statement 3 is incorrect.

www.insightsactivelearn.com 23
Total Marks : 200
Test-3 (Subject)
( INSTA Prelims Test Series 2021 )

https://www.cnbctv18.com/finance/monetary-policy-has-space-but-will-be-used-prudently-says-
rbi-governor-shaktikanta-das-6699211.htm

31. Consider the following statements regarding Deccan Riots


1. The ryots of Deccan region of western India suffered heavy taxation under the Ryotwari
system.
2. As a conciliatory measure with ryots, the Deccan Agriculturists Relief Act was passed in 1879.

Which of the statements given above is/are correct?


A. 1 only
B. 2 only
C. Both 1 and 2
D. Neither 1 nor 2

Correct Answer : C

Answer Justification :

Both the statements are correct.

Deccan Riots:

The ryots of Deccan region of western India suffered heavy taxation under the
Ryotwari system. Here again the peasants found themselves trapped in a vicious network
with the moneylender as the exploiter and the main beneficiary.

Moneylenders were mostly outsiders—Marwaris or Gujaratis. The conditions had worsened


due to a crash in cotton prices after the end of the American Civil War in 1864, the
Government’s decision to raise the land revenue by 50% in 1867, and a succession of bad
harvests.

The Government succeeded in repressing the movement. As a conciliatory measure,


the Deccan Agriculturists Relief Act was passed in 1879. This time also, the modern
nationalist intelligentsia of Maharashtra supported the peasants’ cause.

32. The battle of Hooghly (1759), dealt a crushing blow to the ambitions of

A. Danes
B. Portuguese
C. Dutch
D. French

www.insightsactivelearn.com 24
Total Marks : 200
Test-3 (Subject)
( INSTA Prelims Test Series 2021 )

Correct Answer : C

Answer Justification :

Decline of the Dutch in India:

The Dutch got drawn into the trade of the Malay Archipelago. Further, in the third Anglo-
Dutch War (1672-74), communications between Surat and the new English settlement of
Bombay got cut due to which three homebound English ships were captured in the Bay of
Bengal by the Dutch forces.

The retaliation by the English resulted in the defeat of the Dutch, in the battle of
Hooghly (November 1759), which dealt a crushing blow to Dutch ambitions in India.
Hence, option (c) is correct.

The Dutch were not much interested in empire building in India; their concerns were trade. In
any case, their main commercial interest lay in the Spice Islands of Indonesia from where they
earned a huge profit through business.

33. Who among the following social reformers established the Advaita Ashram at Aluva?

A. K. M. Kesavan
B. T. K. Madhavan
C. Sree Narayana Guru
D. E. M. S. Namboodiripad

Correct Answer : C

Answer Justification :

Sree Narayana Guru founded the Advaita Ashram at Aluva in 1913. This Ashram was
dedicated to a great principle – Om Sahodaryam Sarvatra (all men are equal in the
eyes of God).

In 1921, a Conference of Universal Brotherhood was held at Aluva. In 1924, a


conference of all religions was held there. The Guru stressed the need for a Brahma Vidyalaya
for a comparative study of different religious faiths.

Hence, option (c) is correct.

34. Consider the following statements regarding the Carnatic Wars


1. The First Carnatic War was an extension of the Anglo-French War in Europe which was caused
by the Austrian War of Succession.
www.insightsactivelearn.com 25
Total Marks : 200
Test-3 (Subject)
( INSTA Prelims Test Series 2021 )

2. The Second Carnatic War was an extension of the Anglo-French War in Europe which was
caused by the Seven Years War (1756-63).
3. Third Carnatic War was due to rivalry in India for power, political influence and expansion of
territories.

Which of the statements given above is/are correct?


A. 1 only
B. 1 and 3 only
C. 2 and 3 only
D. 1, 2 and 3

Correct Answer : A

Answer Justification :

The Anglo-French Struggle for Supremacy: the Carnatic Wars

Background of Rivalry:

Though the British and the French came to India for trading purposes, they were ultimately
drawn into the politics of India. Both had visions of establishing political power over the
region. The Anglo-French rivalry in India reflected the traditional rivalry of England and
France throughout their histories;

First Carnatic War (1740-48)

Background:

Carnatic was the name given by the Europeans to the Coromandel coast and its hinterland.
The First Carnatic War was an extension of the Anglo-French War in Europe which
was caused by the Austrian War of Succession.

Result:

The First Carnatic War ended in 1748 when the Treaty of Aix-La Chapelle was signed
bringing the Austrian War of Succession to a conclusion. Under the terms of this treaty,
Madras was handed back to the English, and the French, in turn, got their territories in North
America.

Second Carnatic War (1749-54):

Background:

The background for the Second Carnatic War was provided by rivalry in India. Hence,
statement 2 is incorrect.

Dupleix, the French governor who had successfully led the French forces in the First Carnatic
War, sought to increase his power and French political influence in southern India by

www.insightsactivelearn.com 26
Total Marks : 200
Test-3 (Subject)
( INSTA Prelims Test Series 2021 )

interfering in local dynastic disputes to defeat the English.

Result:

The French authorities, annoyed at the heavy financial losses that Dupleix’s policy involved,
decided to recall him in 1754. Godeheu succeeded Dupleix as the French governor-general in
India. Godeheu adopted a policy

of negotiations with the English and concluded a treaty with them.

Third Carnatic War (1758-63):

Background:

In Europe, when Austria wanted to recover Silesia in 1756, the Seven Years War
(1756-63) started. Britain and France were once again on opposite sides. Hence,
statement 3 is incorrect.

Course of War in India:

In 1758, the French army under Count de Lally captured the English forts of St. David and
Vizianagaram. Now, the English became offensive and inflicted heavy losses on the French
fleet under Admiral D’Ache at Masulipatnam.

Battle of Wandiwash:

The decisive battle of the Third Carnatic War was won by the English on January 22, 1760 at
Wandiwash (or Vandavasi) in Tamil Nadu.

Result and Significance:

The Third Carnatic War proved decisive. Although the Treaty of Peace of Paris (1763)
restored to the French their factories in India, the French political influence disappeared after
the war.

35. Consider the following statements regarding the Electronic Vaccine Intelligence Network (eVIN):
1. It aims to provide real-time information on vaccine stocks and flows, and storage temperatures
across all cold chain points in the country.
2. It is being implemented under National Health Mission (NHM) by Ministry of Health and
Family Welfare.

Which of the statements given above is/are correct?


A. 1 only
B. 2 only
C. Both 1 and 2
D. Neither 1 nor 2

www.insightsactivelearn.com 27
Total Marks : 200
Test-3 (Subject)
( INSTA Prelims Test Series 2021 )

Correct Answer : C

Answer Justification :

Both the statements given above are correct.

The Electronic Vaccine Intelligence Network (eVIN) is an innovative technological


solution aimed at strengthening immunization supply chain systems across the
country. This is being implemented under National Health Mission (NHM) by
Ministry of Health and Family Welfare. eVIN aims to provide real-time information on
vaccine stocks and flows, and storage temperatures across all cold chain points in the
country. This robust system has been used with the requisite customization during the COVID
pandemic for ensuring continuation of the essential immunization services and protecting our
children and pregnant mothers against vaccine preventable diseases.

eVIN combines state-of-the-art technology, a strong IT infrastructure and trained human


resource to enable real time monitoring of stock and storage temperature of the vaccines kept
in multiple locations across the country.

eVIN has reached 32 States and Union Territories (UTs) and will soon be rolled-out in the
remaining States and UTs of Andaman & Nicobar Islands, Chandigarh, Ladakh and Sikkim.

https://pib.gov.in/PressReleasePage.aspx?PRID=1643172

36. Consider the following events


1. Eka Movement
2. Bardoli Satyagraha
3. Tebhaga Movement

Which of the following is the correct chronological sequence of the above events?
A. 2-3-1
B. 1-2-3
C. 1-3-2
D. 2-1-3

Correct Answer : B

Answer Justification :

Eka Movement

Towards the end of 1921, peasant discontentment resurfaced in some northern districts of
the United Provinces—Hardoi, Bahraich, Sitapur. The issues involved were:

www.insightsactivelearn.com 28
Total Marks : 200
Test-3 (Subject)
( INSTA Prelims Test Series 2021 )

(i) high rents—50 per cent higher than the recorded rates;
(ii) oppression of thikadars in charge of revenue collection; and
(iii) practice of share-rents.

Bardoli Satyagraha:

The Bardoli taluqa in Surat district had witnessed intense politicisation after the
coming of Gandhi on the national political scene. The movement sparked off in
January 1926 when the authorities decided to increase the land revenue by 30 per cent.

The Congress leaders were quick to protest and a Bardoli Inquiry Committee was set up to go
into the issue. The committee found the revenue hike to be unjustified. In February 1926,
Vallabhbhai Patel was called to lead the movement.

Tebhaga Movement:

In September 1946, the Bengal Provincial Kisan Sabha gave a call to implement, through
mass struggle, the Flood Commission recommendations of tebhaga—two-thirds’ share—to the
bargardars, the share-croppers also known as bagchasi or adhyar, instead of the one-half
share.

The bargardars worked on lands rented from the jotedars. The communist cadres, including
many urban student militias went to the countryside to organise the bargardars. The central
slogan was “nij khamare dhan tolo”—i.e.,
sharecroppers taking the paddy to their own threshing floor and not to the jotedar’s house, as
before, so as to enforce tebhaga.

37. Consider the following statements regarding Muhammad Shah


1. Muhammad Shah was given the title of ‘Rangeela’ due to his luxurious life-style.
2. He introduced izara system to improve the financial condition of the empire.

Which of the statements given above is/are correct?


A. 1 only
B. 2 only
C. Both 1 and 2
D. Neither 1 nor 2

Correct Answer : A

Answer Justification :

Jahandar Shah (March 1712-February 1713):

With the help of Zulfikar Khan, Jahandar Shah became the emperor. Zulfikar Khan was
appointed prime minister; he introduced izara system to improve the financial condition

www.insightsactivelearn.com 29
Total Marks : 200
Test-3 (Subject)
( INSTA Prelims Test Series 2021 )

of the empire. Jahandar Shah abolished Jaziya.

Hence, statement 2 is incorrect.

Muhammad Shah (1719-48):

After the death of Rafiud-Daula, Raushan Akhtar became the choice of the Sayyid Brothers.
Muhammad Shah, as he came to be known in history, was given the title of ‘Rangeela’
due to his luxurious life-style.

Muhammad Shah, with the help of Nizam-ul-Mulk, killed the Sayyid Brothers. In 1724, Nizam-
ul-Mulk became the wazir and founded the independent state of Hyderabad.

In 1737, Baji Rao I, the Maratha Peshwa invaded Delhi with a small army of 500 horsemen. In
1739, Nadir Shah defeated the Mughals in the Battle of Karnal and later imprisoned
Muhammad Shah and annexed areas west of the Indus into the Persian Empire.

38. Which of the following is/are can be considered as causes of the Sepoy Revolt 1857:
1. The annexation of Jhansi by Dalhousie employing the Doctrine of Lapse.
2. Oppressive land revenue system.
3. Spread of the news that the fat of cows and pigs was used in the greased cartridges.

Select the correct answer using the code given below:


A. 1 and 2 only
B. 2 and 3 only
C. 1 and 3 only
D. 1, 2 and 3

Correct Answer : D

Answer Justification :

All the statements given above are correct.

Revolt of 1857:

The Indian Rebellion of 1857 was a major, but ultimately unsuccessful, uprising in India in
1857–58 against the rule of the British East India Company, which functioned as a sovereign
power on behalf of the British Crown.

Causes of Rebellion:

(i) Territorial Aggrandizement: The annexation of Oudh and Jhansi by Dalhousie


employing the Doctrine of Lapse and the humiliating treatment meted out to Nana Sahib,

www.insightsactivelearn.com 30
Total Marks : 200
Test-3 (Subject)
( INSTA Prelims Test Series 2021 )

the last Peshwa’s adopted son produced much dissatisfaction.

(ii) Oppressive Land Revenue System: The British treated land revenue as a rent and not a
tax. This meant that revenue was extracted whether the land was cultivated or not, and at the
same rate. The prices of agricultural commodities continued to crash throughout the first half
of nineteenth century and in the absence of any remission or relief from the colonial state,
small and marginal farmers as well as cultivating tenants were subject to untold misery

(iii) Alienation of Muslim Aristocracy and Intelligentsia: English language and western
education pushed the Muslim intelligentsia into insignificance. The abolition of Persian
language in the law courts and admission into public service by examination decreased the
Muslim’s chances of official employment

(iv) Religious Sentiments: The Act of 1856 providing for enrolment of high caste men as
sepoys in the Bengal army stipulated that future recruits give up martial careers or their caste
scruples. This apart, acts such as the abolition of sati, legalization of remarriage of Hindu
widows, prohibition of infanticide were viewed as interference in religious beliefs. In 1850, to
the repugnance of orthodox Hindus, the Lex Loci Act was passed permitting converts to
Christianity to retain their patrimony (right to inherit property from parents or ancestors)

The religious sentiments of the sepoys – Hindus and Muslims – were outraged when
information spread that the fat of cows and pigs was used in the greased cartridges.
The Indian sepoys were to bite them before loading the new Enfield rifle. This was viewed as a
measure to convert people to Christianity.

39. Consider the following statements


1. Under the reign of Alamgir II, the Third Battle of Panipat took place.
2. Under the reign of Shah Alam II, two decisive battles the Battle of Plassey and the Battle of
Buxar took place.

Which of the statements given above is/are correct?


A. 1 only
B. 2 only
C. Both 1 and 2
D. Neither 1 nor 2

Correct Answer : D

Answer Justification :

Alamgir II (1754-1758):

Alamgir II was a grandson of Jahandar Shah. Ahmed Shah Abdali, the Iranian invader, reached

www.insightsactivelearn.com 31
Total Marks : 200
Test-3 (Subject)
( INSTA Prelims Test Series 2021 )

Delhi in January 1757. During his reign, the Battle of Plassey was fought in June 1757.

Hence, statement 1 is incorrect.

Shah Alam II (1759-1806):

His reign saw two decisive battles—the Third Battle of Panipat (1761) and the Battle of
Buxar (1764). Hence, statement 2 is incorrect.

In 1765, according to the terms of Treaty of Allahabad (August 1765), he was taken under the
East India Company’s protection and resided at Allahabad. He also issued a farman granting to
the Company in perpetuity the Diwani (the right to collect revenue) of Bengal, Bihar and
Orissa.

40. Consider the following statements regarding the Pokkali Rice:


1. It is known for its salinity tolerance.
2. It is grown in the coastal districts of Kerala.

Which of the statements given above is/are correct?


A. 1 only
B. 2 only
C. Both 1 and 2
D. Neither 1 nor 2

Correct Answer : C

Answer Justification :

Both the statements given above are correct.

The pokkali variety of rice is known for its saltwater resistance and flourishes in the
rice paddies of coastal Kerala districts.

The uniqueness of the rice has brought it the Geographical Indication (GI) tag and is the
subject of continuing research.

The organically-grown Pokkali is famed for its peculiar taste and its high protein
content.

Majority of Pokkali land lie between the Vembanad Lake and the Arabian Sea coast of
Kerala state

https://www.thehindu.com/news/national/other-states/pokkali-rice-seedlings-travel-from-kerala
-to-the-sunderbans/article32278833.ece

www.insightsactivelearn.com 32
Total Marks : 200
Test-3 (Subject)
( INSTA Prelims Test Series 2021 )

41. Consider the following statements regarding the system of Dual Government in Bengal:
1. Robert Clive introduced the dual system of government.
2. The Nizamat rights, i.e., police and judicial functions, came under the control of the Nawab.
3. The Diwani rights, i.e., collecting revenues, came under the control of the Company.

Which of the statements given above is/are correct?


A. 1 and 2 only
B. 1 and 3 only
C. 2 and 3 only
D. 1, 2 and 3

Correct Answer : B

Answer Justification :

Dual Government in Bengal (1765-72)

After the battle of Buxar, the East India Company became the real masters of Bengal. Robert
Clive introduced the dual system of government, i.e., the rule of the two—the
Company and the Nawab—in Bengal in which both the diwani, i.e., collecting
revenues, and nizamat, i.e., police and judicial functions, came under the control of
the Company. Hence, statement 2 is incorrect.

The Company exercised diwani rights as the diwan and the nizamat rights through its right to
nominate the deputy subahdar. The Company acquired the diwani functions from the emperor
and nizamat functions from the subahdar of Bengal.

The dual system led to an administrative breakdown and proved disastrous for the people of
Bengal. Neither the Company nor the Nawab cared for administration and public welfare.
Warren Hastings did away with the dual system in 1772.

42. Which of the following personalities participated during Swadeshi Movement?


1. Ashwini Coomar Banerjea
2. Prabhat Kumar Roy
3. Subramaniya Siva

Select the correct answer using the code given below


A. 1 and 2 only
B. 1, 2 and 3
C. 1 and 3 only
D. 2 and 3 only

Correct Answer : B

www.insightsactivelearn.com 33
Total Marks : 200
Test-3 (Subject)
( INSTA Prelims Test Series 2021 )

Answer Justification :

All the above personalities participated in Swadeshi Movement.

Workers participated in wider political issues. Strikes were organised by Ashwini Coomar
Banerjea, Prabhat Kumar Roy.

Chaudhuri, Premtosh Bose and Apurba Kumar Ghosh. These strikes were organised in
government press, railways and the jute industry.

There were attempts to form trade unions but these were not very successful.
Subramaniya Siva and Chidambaram Pillai led strikes in Tuticorin and Tirunelvelli and
were arrested.

43. Who among the following social reformers started the newspaper ‘Rast-Goftar’?

A. Anand Mohan Bose


B. Surendranath Banerjea
C. Dadabhai Naoroji
D. Raja Rammohan Roy

Correct Answer : C

Answer Justification :

Rast Goftar ("The Truth Teller") was an Anglo-Gujarati paper operating in Bombay
that was started in 1854 by Dadabhai Naoroji and Kharshedji Cama and championed
social reform among Parsis in Western India.

Hence, option (c) is correct.

44. Consider the following pairs


Anglo-Maratha War Treaties
1. First Anglo-Maratha War: Treaty of Salbai
2. Second Anglo Maratha War: Treaty of Bassein
3. Third Anglo-Maratha War: Treaty of Poona

Which of the pairs given above is/are correctly matched?


A. 1 and 2 only
B. 1 and 3 only
C. 2 and 3 only
D. 1, 2 and 3

www.insightsactivelearn.com 34
Total Marks : 200
Test-3 (Subject)
( INSTA Prelims Test Series 2021 )

Correct Answer : D

Answer Justification :

All the pairs given above are correctly matched.

First Anglo-Maratha War (1775-82):

Treaty of Salbai (1782): End of the First Phase of the Struggle

Sindhia proposed a new treaty between the Peshwa and the English, and the Treaty of Salbai
was signed in May 1782; it was ratified by Hastings in June 1782 and by Phadnavis in
February 1783. The treaty guaranteed peace between the two sides for twenty years.

Second Anglo Maratha War (1803-1805):

In 1804, Jashwantrao Holkar made an attempt to form a coalition of Indian rulers to fight
against the English. But his attempt proved unsuccessful. The Marathas were defeated,
reduced to British vassalage and isolated from one another.

[(i) Defeat of Bhonsle (December 17, 1803, Treaty of Devgaon); (ii) Defeat of Sindhia
(December 30, 1803, Treaty of Surajianjangaon); and (iii) Defeat of Holkar(1806, Treaty of
Rajpurghat)].

Treaty of Bassein (1802):

Under the treaty, the Peshwa agreed:

(i) to receive from the Company a native infantry (consisting of not less than 6,000 troops),
with the usual proportion of field artillery and European artillery men attached, to be
permanently stationed in his territories;

(ii) to cede to the Company territories yielding an income of Rs 26 lakh;

(iii) to surrender the city of Surat;

(iv) to give up all claims for chauth on the Nizam’s dominions;

(v) to accept the Company’s arbitrate in all differences between him and the Nizam or the
Gaekwad;

(vi) not to keep in his employment Europeans of any nation at war with the English; and

(vii) to subject his relations with other states to the control of the English.

www.insightsactivelearn.com 35
Total Marks : 200
Test-3 (Subject)
( INSTA Prelims Test Series 2021 )

Third Anglo-Maratha War (1817-19):

The Treaty of Bassein, described as “a treaty with a cipher (the Peshwa)”, wounded the
feelings of the other Maratha leaders. They saw the treaty as an absolute surrender of
independence.

Result:

The Peshwa was defeated at Khirki, Bhonsle at Sitabuldi, and Holkar at Mahidpur. Some
important treaties were signed. These were:

● June 1817, Treaty of Poona, with Peshwa.

● November 1817, Treaty of Gwalior, with Sindhia.

● January 1818, Treaty of Mandasor, with Holkar.

In June 1818, the Peshwa finally surrendered and the Maratha confederacy was dissolved. The
peshwaship was abolished. Peshwa Bajirao became a British retainer at Bithur near Kanpur.
Pratap Singh, a lineal descendant of Shivaji, was made ruler of a small principality, Satara,
formed out of the Peshwa’s dominions.

45. Consider the following statements regarding the Swadesh Darshan Scheme:
1. It was launched by the Ministry of Culture.
2. It is a central sector scheme.
3. It aims to develop theme-based tourist circuits in the country.

Which of the statements given above is/are correct?


A. 1 and 2 only
B. 3 only
C. 2 and 3 only
D. 1, 2 and 3

Correct Answer : C

Answer Justification :

Ministry of Tourism (MoT) launched the Swadesh Darshan Scheme (Central Sector

www.insightsactivelearn.com 36
Total Marks : 200
Test-3 (Subject)
( INSTA Prelims Test Series 2021 )

Scheme)– for integrated development of theme based tourist circuits in the country in
2014-15. Hence, statement 1 is incorrect. However, statement 2 and statement 3 are
correct.

This scheme is envisioned to synergise with other Government of India schemes like Swachh
Bharat Abhiyan, Skill India, Make in India etc. with the idea of positioning the tourism sector
as a major engine for job creation, driving force for economic growth, building synergy with
various sectors to enable tourism to realise its potential.

Features of Swadesh Darshan Scheme:

The scheme is 100% centrally funded for the project components undertaken for public
funding.
To leverage the voluntary funding available for Corporate Social Responsibility (CSR)
initiatives of Central Public Sector Undertakings and corporate sector.
Funding of individual project will vary from state to state and will be finalised on the
basis of detailed project reports prepared by PMC (Programme Management
Consultant).
A National Steering Committee (NSC)will be constituted with Minister in charge of M/O
Tourism as Chairman, to steer the mission objectives and vision of the scheme.
A Mission Directorate headed by the Member Secretary, NSC as a nodal officer will help
in identification of projects in consultation with the States/ UTs governments and other
stake holders.
PMC will be a national level consultant to be appointed by the Mission Directorate.

The Ministry of Tourism, under the Swadesh Darshan scheme is developing thematic circuits
in the country in planned and prioritized manner. Under the scheme fifteen thematic
circuits have been identified for development namely; North-East Circuit, Buddhist
Circuit, Himalayan Circuit, Coastal Circuit, Krishna Circuit, Desert Circuit, Tribal
Circuit, Eco Circuit, Wildlife Circuit, Rural Circuit, Spiritual Circuit, Ramayana
Circuit, Heritage Circuit, Sufi Circuit, and Tirthankara Circuit.

https://pib.gov.in/PressReleasePage.aspx?PRID=1643290

https://pib.gov.in/PressReleasePage.aspx?PRID=1601785

46. The Treaty of ‘Eternal Friendship’ was signed by British with

A. Afghanistan
B. Sindh
C. Bhutan
D. Nepal

www.insightsactivelearn.com 37
Total Marks : 200
Test-3 (Subject)
( INSTA Prelims Test Series 2021 )

Correct Answer : B

Answer Justification :

Treaty of ‘Eternal Friendship’ with Sindh

In June 1807, the alliance of Tilsit with Alexander I of Russia was joined by Napoleon
Bonaparte. The alliance had as one of its conditions a combined invasion of India by the land
route. Now the British wanted to create a barrier between Russia and British India. To achieve
this, Lord Minto sent

three delegations under the leadership of various prominent persons to forge alliances.
Accordingly, Metcalfe was sent to Lahore, Elphinstone to Kabul and Malcolm to Teheran.
Sindh was visited by Nicholas Smith who met the Amirs to conclude a defensive arrangement.
After negotiations, the Amirs agreed to a treaty—their first-ever treaty with the
English. After professing eternal friendship, both sides agreed to exclude the French
from Sindh and to exchange agents at each other’s court. The treaty was renewed in
1820 with the addition of an article excluding the Americans and resolving some border
disputes on the side of Kachch after the final defeat of the Maratha confederacy in 1818.

Hence, option (b) is correct.

47. Consider the following statements regarding the Lord Macaulay’s Minute
1. It recommended for the education of masses.
2. Lord Macaulay held the view that Indian learning was inferior to European learning.

Which of the statements given above is/are correct?


A. 1 only
B. 2 only
C. Both 1 and 2
D. Neither 1 nor 2

Correct Answer : B

Answer Justification :

Lord Macaulay’s Minute (1835):

The famous Lord Macaulay’s Minute settled the row in favour of Anglicists—the limited
government resources were to be devoted to teaching of Western sciences and literature
through the medium of English language alone.

Lord Macaulay held the view that “Indian learning was inferior to European
learning”—which was true as far as physical and social sciences in the contemporary stage

www.insightsactivelearn.com 38
Total Marks : 200
Test-3 (Subject)
( INSTA Prelims Test Series 2021 )

were concerned. Hence Statement 2 is correct.

The government soon made English as the medium of instruction in its schools and
colleges and opened a few English schools and colleges instead of a large number of
elementary schools, thus neglecting mass education. Hence Statement 1 is incorrect.

The British planned to educate a small section of upper and middle classes, thus creating a
class “Indian in blood and colour but English in tastes, in opinions, in morals and in intellect”
who would act as interpreters between the government and masses and would enrich the
vernaculars by which knowledge of Western sciences and literature would reach the masses.
This was called the ‘downward filtration theory’.

48. Consider the following statements regarding the Prarthana Samaj:


1. It was founded in 1867 in Bombay by Atmaram Pandurang.
2. The Prarathana Samaj continued its work mainly through educational work directed at women
and workers at the lower level.

Which of the statements given above is/are correct?


A. 1 only
B. 2 only
C. Both 1 and 2
D. Neither 1 nor 2

Correct Answer : C

Answer Justification :

Both the statements given above are correct.

An off-shoot of the Brahmo Samaj, the Prarthana Samaj, was founded in 1867 in
Bombay by Atmaram Pandurang (1823– 98). The Prarthana Samaj as an organization
never had any great influence but its members, like M. G. Ranade (1852-1901), R. G.
Bhandarkar, and K.T. Telang, were among the great leaders of nineteenthcentury Maharashtra
and they became the founders of the social reform movement in later years.

Prarthana Samaj was similar to Brahmo Samaj, but it was consciously linked with the
bhakti tradition of the Maharashtrian saints. The Prarathana Samaj continued its
work mainly through educational work directed at women and workers at the lower
level. It concentrated on social reforms like inter-dining, intermarriage, remarriage of widows,
and uplift of women and depressed classes.

49. Consider the following statements regarding Anglo-Sikh Wars


1. The Anglo-Sikh wars gave the two sides a mutual respect for each other’s fighting prowess.

www.insightsactivelearn.com 39
Total Marks : 200
Test-3 (Subject)
( INSTA Prelims Test Series 2021 )

2. The first Anglo-Sikh War ended with Treaty of Amritsar.


3. John Lawrence became the first chief commissioner of Punjab.

Which of the statements given above is/are correct?


A. 1 only
B. 1 and 3 only
C. 2 and 3 only
D. 1, 2 and 3

Correct Answer : B

Answer Justification :

First Anglo-Sikh War (1845-46)

Causes:

The outbreak of the first of the Anglo-Sikh wars has been attributed to the action of the Sikh
army crossing the River Sutlej on December 11, 1845. This was seen as an aggressive
manoeuvre that provided the English with the justification to declare war.

Treaty of Lahore (March 8, 1846):

The end of the first Anglo-Sikh War forced the Sikhs to sign a humiliating treaty on
March 8, 1846. Hence, statement 2 is incorrect.

Second Anglo-Sikh War (1848-49):

Causes:

The defeat in the first Anglo-Sikh War and the provisions of the treaties of Lahore and
Bhairowal were highly humiliating for the Sikhs. Inhuman treatment meted out to Rani Jindan,
who was sent to Benares as a pensioner, added to the resentment of the Sikhs.

Result:

At the end of the war came:

www.insightsactivelearn.com 40
Total Marks : 200
Test-3 (Subject)
( INSTA Prelims Test Series 2021 )

● surrender of the Sikh army and Sher Singh in 1849;

● annexation of Punjab; and for his services the Earl of Dalhousie was given the thanks of the
British Parliament and a promotion in the peerage, as Marquess;

● setting up of a three-member board to govern Punjab, comprising of the Lawrence brothers


(Henry and John) and Charles Mansel.

In 1853 the board was nullified and Punjab was placed under a chief commissioner.
John Lawrence became the first chief commissioner.

Significance of the Anglo-Sikh Wars

The Anglo-Sikh wars gave the two sides a mutual respect for each other’s fighting
prowess. The Sikhs were to fight loyally on the British side in the Revolt of 1857 and
in many other campaigns and wars until the Indian independence in 1947.

50. Consider the following statements regarding the Teesta River:


1. It originates in the Sikkim Himalayas.
2. The Rangeet River is a main tributary of Teesta River.
3. It forms the border between Sikkim and West Bengal.

Which of the statements given above is/are correct?


A. 1 and 2 only
B. 2 and 3 only
C. 1 and 3 only
D. 1, 2 and 3

Correct Answer : D

Answer Justification :

All the statements given above are correct.

The literal meaning of the word Teesta is Trishna (desire) which never ends. The legend of the
Teesta River is mentioned in KalikaPurana. Among the other rivers of northern India, Teesta
River is also called as younger daughter of Himalaya.

The Teesta River originates from the Pahunri (or Teesta Kangse) glacier above 7,068
m, and flows southward through gorges and rapids in the Sikkim Himalaya. The river
then flows past the town of Rangpo where the Rangpo River joins, and where it forms
the border between Sikkim and West Bengal up to Teesta Bazaar. Just before the
Teesta Bridge, where the roads from Kalimpong and Darjeeling join, the river is met
by its main tributary, the Rangeet River. At this point, it changes course southwards

www.insightsactivelearn.com 41
Total Marks : 200
Test-3 (Subject)
( INSTA Prelims Test Series 2021 )

flowing into West Bengal. The river then goes merging up with the Brahmaputra River after it
bifurcates the city of Jalpaiguri and flows just touching Cooch Behar district at Mekhliganj and
moves to Fulchori in Bangladesh. Teesta River ultimately drains into Brahmaputra at
Teestamukh Ghat in Kamarjani- Bahadurabad in Rangpur district of Bangladesh. Teesta and
most of its tributaries are flashy mountain rivers and carry boulders and considerable quantity
of sediment. The flow is turbulent and characterised by high velocities.

The Teesta basin in India extends over an area of 9,855 sq.km, which is nearly 0.28%
of the total geographical area of the country. The basin lies in the states of Sikkim
(72.43%) and West Bengal (27.57%). Teesta is a 414 km long river with total drainage area
of 12,540 sq.km,flowing through India and Bangladesh.

https://indianexpress.com/article/explained/china-twist-in-teesta-river-project-challenge-65633
98/

51. Consider the following statements regarding the Subsidiary System:


1. Satara was the first state which was brought under Wellesley’s Subsidiary System.
2. Wellesley reversed the non-intervention policy of his predecessor and formulated the
‘Subsidiary Alliance’.
3. The Indian state was called ‘the protected state’ and the British hereinafter were referred to
as ‘the paramount power’.

Which of the statements given above is/are correct?


A. 1 and 2 only
B. 1 and 3 only
C. 2 and 3 only
D. 1, 2 and 3

Correct Answer : C

Answer Justification :

Wellesley was also thoroughly convinced that only a strong British power in India could reduce
and control the existing tyranny and corruption in Indian states. Therefore, he reversed the
nonintervention policy of his predecessor and formulated his master plan namely the
‘Subsidiary Alliance’.

The Subsidiary System:

The predecessors of Wellesley concluded alliances with Indian princes like the Nawab of Oudh
and the Nizam of Hyderabad. They received subsidies from the Indian rulers for the
maintenance of British troops, which were used for the protection of respective Indian states.
Wellesley enlarged and consolidated the already existing system. However, his originality was

www.insightsactivelearn.com 42
Total Marks : 200
Test-3 (Subject)
( INSTA Prelims Test Series 2021 )

revealed in its application.

Main Features of Subsidiary Alliance:

1. Any Indian ruler who entered into the subsidiary alliance with the British had to maintain a
contingent of British troops in his territory. It was commanded by a British officer. The Indian
state was called ‘the protected state’ and the British hereinafter were referred to as
‘the paramount power’. It was the duty of the British to safeguard that state from external
aggression and to help its ruler maintain internal peace. The protected state should give some
money or give part of its territory to the British to support the subsidiary force.

2. The protected state should cut off its connection with European powers other than the
English and with the French in particular. The state was also forbidden to have any political
contact even with other Indian powers without the permission of the British.

3. The ruler of the protected state should keep a British Resident at his court and disband his
own army. He should not employ Europeans in his service without the sanction of the
paramount power.

4. The paramount power should not interfere in the internal affairs of the protected state.

Hyderabad: Hyderabad was the first state which was brought under Wellesley’s
Subsidiary System in 1798. Hence, statement 1 is incorrect.

52. With reference to the education policies of British, consider the following statements
1. Wood’s Despatch was considered as the “Magna Carta of English Education in India”
2. Wood’s Despatch recommended English as the medium of instruction at all levels of the
education.

Which of the statements given above is/are correct?


A. 1 only
B. 2 only
C. Both 1 and 2
D. Neither 1 nor 2

Correct Answer : A

Answer Justification :

In 1854, Charles Wood prepared a despatch on an educational system for India.


Considered the “Magna Carta of English Education in India”, this document was the
first comprehensive plan for the spread of education in India. Hence Statement 1 is correct.

1. It asked the government of India to assume responsibility for education of the masses, thus
repudiating the ‘downward filtration theory’, at least on paper.

www.insightsactivelearn.com 43
Total Marks : 200
Test-3 (Subject)
( INSTA Prelims Test Series 2021 )

2. It systematised the hierarchy from vernacular primary schools in villages at the bottom,
followed by Anglo-Vernacular High Schools and an affiliated college at the district level, and
affiliating universities in the presidency towns of Calcutta, Bombay and Madras.

3. It recommended English as the medium of instruction for higher studies and vernaculars at
school level. Hence Statement 2 is incorrect.

4. It laid stress on female and vocational education, and on teachers’ training.

53. Which of the following Acts provided for the right to ask questions and discuss the budget?

A. Indian Councils Act, 1892


B. Charter Act, 1833
C. Charter Act, 1813
D. Indian Councils Act, 1861

Correct Answer : A

Answer Justification :

Features of the Act of 1892:

1. It increased the number of additional (non-official) members in the Central and provincial
legislative councils, but maintained the official majority in them.

2. It increased the functions of legislative councils and gave them the power of
discussing the budget and addressing questions to the executive. Hence, option (a) is
correct.

3. It provided for the nomination of some non-official members of the

(a) Central Legislative Council by the viceroy on the recommendation of the provincial
legislative councils and the Bengal Chamber of Commerce, and

(b) that of the Provincial legislative councils by the Governors on the recommendation of the
district boards, municipalities, universities, trade associations, zamindars and chambers.

‘The act made a limited and indirect provision for the use of election in filling up some of the
non-official seats both in the Central and provincial legislative councils. The word “election”
was, however, not used in the act. The process was described as nomination made on the
recommendation of certain bodies

54. Consider the following statements regarding the Charter Act of 1833
1. The liberal and utilitarian philosophy of Bentham was made popular by the provisions of this

www.insightsactivelearn.com 44
Total Marks : 200
Test-3 (Subject)
( INSTA Prelims Test Series 2021 )

Act.
2. A Law Member was appointed to the Governor-General’s Council and T. B. Macaulay was the
first Law Member.

Which of the statements given above is/are correct?


A. 1 only
B. 2 only
C. Both 1 and 2
D. Neither 1 nor 2

Correct Answer : C

Answer Justification :

The Charter Act of 1833 was a significant constitutional instrument defining the scope and
authority of the East India Company. The liberal and utilitarian philosophy of Bentham
was made popular by the provisions of this Act. Following were the important provisions:

(i) The English East India Company ceased to be a commercial agency in India. In other words,
it would function hereafter as the political agent for the Crown.

(ii) The Governor-General of Fort William was hereafter called ‘the Governor- General of
India’. Thus, Bentinck was the first Governor-General of India’.

(iii) A Law Member was appointed to the Governor-General’s Council. T. B. Macaulay


was the first Law Member of the Governor- General-in-Council.

(iv) The Act categorically stated ‘that no native of India, nor any natural born subject of His
Majesty, should be disabled from holding any place, office, or employment, by reason of his
religion, place of birth, descent or colour”. It was this enactment which laid the foundation for
the Indianisation of public services.

55. Consider the following statements regarding the Prime Minister’s Employment Generation Programme
(PMEGP):
1. It is a credit linked subsidy programme for generation of employment opportunities through
establishment of micro enterprises in rural areas only to arrest the forced urban migration.
2. It is central sector scheme to be administered by the Ministry of Rural Development.
3. The Scheme is implemented by the National Bank for Agriculture and Rural Development
(NABARD).

Which of the statements given above is/are correct?


A. 1 only
B. 2 only
C. 3 only
D. None

www.insightsactivelearn.com 45
Total Marks : 200
Test-3 (Subject)
( INSTA Prelims Test Series 2021 )

Correct Answer : D

Answer Justification :

None of the statements given above is correct.

Prime Minister Employment Generation Programme (PMEGP):

Government of India has approved the introduction of a new credit linked subsidy
programme called Prime Minister‟s Employment Generation Programme (PMEGP) by
merging the two schemes that were in operation till 31.03.2008 namely Prime
Minister‟s Rojgar Yojana (PMRY) and Rural Employment Generation Programme
(REGP) for generation of employment opportunities through establishment of micro
enterprises in rural as well as urban areas.

PMEGP will be a central sector scheme to be administered by the Ministry of Micro,


Small and Medium Enterprises (MoMSME). The Scheme will be implemented by
Khadi and Village Industries Commission (KVIC), a statutory organization under the
administrative control of the Ministry of MSME as the single nodal agency at the National
level. At the State level, the Scheme will be implemented through State KVIC Directorates,
State Khadi and Village Industries Boards (KVIBs) and District

Industries Centres (DICs) and banks. The Government subsidy under the Scheme will be
routed by KVIC through the identified Banks for eventual distribution to the beneficiaries /
entrepreneurs in their Bank accounts.

Objectives:

(i) To generate employment opportunities in rural as well as urban areas of the country
through setting up of new self-employment ventures/projects/micro enterprises.

(ii) To bring together widely dispersed traditional artisans/ rural and urban unemployed youth
and give them self-employment opportunities to the extent possible, at their place.

(iii) To provide continuous and sustainable employment to a large segment of traditional and
prospective artisans and rural and urban unemployed youth in the country, so as to help arrest
migration of rural youth to urban areas.

(iv) To increase the wage earning capacity of artisans and contribute to increase in the growth
rate of rural and urban employment.

https://www.business-standard.com/article/news-cm/implementation-of-pmegp-projects-record
s-44-jump-in-2020-120082100409_1.html

https://pib.gov.in/PressReleasePage.aspx?PRID=1647366

www.insightsactivelearn.com 46
Total Marks : 200
Test-3 (Subject)
( INSTA Prelims Test Series 2021 )

56. Consider the following statements regarding the administrative reforms by Bentinck
1. In the military department, he abolished the system of double batta.
2. In the judicial department, he abolished the provincial courts of appeal established by
Cornwallis.
3. Introduction of English in the lower courts and higher courts in the place of Persian.

Which of the statements given above is/are correct?


A. 1 and 2 only
B. 1 and 3 only
C. 2 and 3 only
D. 1, 2 and 3

Correct Answer : A

Answer Justification :

Bentinck reduced the salaries and allowances of all officers and additional staff were removed.
In the military department, he abolished the system of double batta. (Batta was an
allowance to troops on active service.) By these financial reforms at the time of his departure,
he left the treasury with a surplus of Rs.1.5 millions.

Administrative Reforms:

Bentinck’s administrative reforms speak of his political maturity and wisdom. In the judicial
department he abolished the provincial courts of appeal established by Cornwallis.
They were largely responsible for the huge arrears of cases. This step was readily accepted by
the Directors since it cut down their expenditure. Another good measure of Bentinck was
the introduction of local languages in the lower courts and English in the higher
courts in the place of Persian. Hence, statement 3 is incorrect. Even in matters of
revenue Bentinck left his mark. He launched the revenue settlements of the North West
Province under the control of R.M. Bird. This settlement was for a period of 30 years and it
was made either with the tillers of the soil, or with the landowners.

57. Consider the following statements regarding the education policies of British
1. Mass education was largely neglected leading to widespread illiteracy.
2. Education became a monopoly of upper and richer classes and city dwellers.
3. Emphasis was given on scientific and technical education.

Which of the statements given above is/are correct?


A. 3 only
B. 2 only
C. 1 and 2 only
D. None

www.insightsactivelearn.com 47
Total Marks : 200
Test-3 (Subject)
( INSTA Prelims Test Series 2021 )

Correct Answer : C

Answer Justification :

Evaluation of British Policy on Education

The British wanted to use modern education to strengthen the foundations of their political
authority in India.

1. Traditional system of Indian learning gradually declined for want of support, and especially
after 1844 when it was declared that applicants for government employment should possess
knowledge of English.

2. Mass education was neglected leading to widespread illiteracy (1911—84 per cent
and in 1921—92 per cent) which created a wide linguistic and cultural gulf between the
educated few and the masses. Hence Statement 1 is correct.

3. Since education was to be paid for, it became a monopoly of upper and richer classes
and city dwellers. Hence Statement 2 is correct.

4. There was an almost total neglect of women’s education because (i) the Government did not
want to arouse wrath of orthodox sections; and (ii) it had no immediate utility for the colonial
rule.

5. Scientific and technical education was by and large neglected. By 1857 there were
only three medical colleges at Calcutta, Bombay and Madras, and only one good engineering
college at Roorkee which was open only to Europeans and Eurasians. Hence, statement 3 is
incorrect.

58. Consider the following statements regarding Gopal Hari Deshmukh:


1. He was popularly known by the name ‘Lokahitawadi’.
2. He started a weekly Hitechchhu in both Gujarati and English.

Which of the statements given above is/are correct?


A. 1 only
B. 2 only
C. Both 1 and 2
D. Neither 1 nor 2

www.insightsactivelearn.com 48
Total Marks : 200
Test-3 (Subject)
( INSTA Prelims Test Series 2021 )

Correct Answer : C

Answer Justification :

Both the statements given above are correct.

Gopalhari Deshmukh (1823-1892) was a social reformer and rationalist from


Maharashtra. He held the post of a judge under British raj, but wrote for a weekly
Prabhakar under the pen name of Lokahitawadi on social reform issues.

He attacked Hindu orthodoxy and supported social and religious equality.

He wrote against the evils of the caste system. He said, “If religion does not sanction
social reform, then change religion.”

He started a weekly, Hitechhu in both Gujarati and English, and also played a leading
role in founding the periodicals, Gyan Prakash, Indu Prakash and Lokahitawadi. He wrote
Panipat war, Kalyog, Jatibhed, Lankecha Itihas.

He also started "Gujarati Budhhi-Wardhak Sabha". He established in Ahmedabad a


branch of Prarthana Samaj, and also founded an institute promoting remarriages of widows,
and invigorated Gujarat Vernacular Society.

59. Consider the following statements regarding the policy of the Doctrine of Lapse
1. Dalhousie maintained that there was a difference in principle between the right to inherit
private property and the right to govern.
2. According to the Hindu Law, one can adopt a son in case of no male heir to inherit the
property.

Which of the statements given above is/are correct?


A. 1 only
B. 2 only
C. Both 1 and 2
D. Neither 1 nor 2

Correct Answer : C

Answer Justification :

Policy of Doctrine of Lapse:

Dalhousie took advantage of every opportunity to acquire territory by peaceful means. The
East India Company was rapidly becoming the predominant power in India. It had concluded

www.insightsactivelearn.com 49
Total Marks : 200
Test-3 (Subject)
( INSTA Prelims Test Series 2021 )

alliances with Indian rulers. It promised to support them and their heirs in return for various
concessions. Although this type of agreement favoured the British, Dalhousie sought to acquire
even more power. According to the Hindu Law, one can adopt a son in case of no male
heir to inherit the property. The question arose whether a Hindu ruler, holding his state
subordinate to the paramount power, could adopt a son to succeed his kingdom. It was
customary for a ruler without a natural heir to ask the British Government whether he could
adopt a son to succeed him. According to Dalhousie, if such permission was refused by the
British, the state would “lapse” and thereby become part of the British India. Dalhousie
maintained that there was a difference in principle between the right to inherit
private property and the right to govern. This principle was called the Doctrine of
Lapse.

The Doctrine of Lapse was applied by Dalhousie to Satara and it was annexed in 1848. Jhansi
and Nagpur were annexed in 1854. As a result of these annexations, a large part of the Central
Provinces came under the British rule. The new province was governed by a Chief
Commissioner from 1861.

60. Which among the following peaks is the highest peak of Aravalli Hills?

A. Guru Shikar
B. Dhupgarh
C. Arma Konda
D. Parasnath

Correct Answer : A

Answer Justification :

The Aravalli Range is a mountain range in Northwestern India, running


approximately 692 km in a south-west direction, starting near Delhi, passing through
southern Haryana and Rajasthan, and ending in Gujarat. The highest peak is Guru
Shikhar at 1,722 metres.

Hence, option (a) is correct.

61. The Policy of Masterly Inactivity was started by:

A. Lord Canning
B. Lord Auckland
C. Lord Lytton
D. John Lawrence

www.insightsactivelearn.com 50
Total Marks : 200
Test-3 (Subject)
( INSTA Prelims Test Series 2021 )

Correct Answer : D

Answer Justification :

John Lawrence and the Policy of Masterly Inactivity:

John Lawrence (1864-1869) started a policy of masterly inactivity which was a reaction to the
disasters of the First Afghan War and an outcome of practical common sense and an intimate
knowledge of the frontier problem and of Afghan passion for independence. Even when Dost
Mohammed died in 1863, there was no interference in the war of succession. Lawrence’s
policy rested on the fulfilment of two conditions—(i) that the peace at the frontier was not
disturbed, and (ii) that no candidate in civil war sought foreign help. And as Sher Ali
established himself on the throne, Lawrence tried to cultivate friendship with him.

Hence, option (d) is correct.

Lytton and the Policy of Proud Reserve:

Lytton, a nominee of the Conservative government under Benjamin Disraeli (1874-80), became
the Viceroy of India in 1876. He started a new foreign policy of ‘proud reserve’, which was
aimed at having scientific frontiers and safeguarding ‘spheres of influence’. According to
Lytton, the relations with Afghanistan could no longer be left ambiguous.

62. With reference to the Development of Vernacular Education during British times, consider the following
statements
1. Lord Dalhousie expressed strong opinion in favor of English education.
2. The Hunter Commission held that State should make special efforts for extension and
improvement of vernacular education.

Which of the statements given above is/are correct?


A. 1 only
B. 2 only
C. Both 1 and 2
D. Neither 1 nor 2

Correct Answer : B

Answer Justification :

Development of Vernacular Education:

During the early 19th century vernacular education was in a sorry state of affairs. It was
mostly dependent on contributions from wealthy zamindars. 1835, 1836, 1838: William Adam’s
reports on vernacular education in Bengal and Bihar pointed out defects in the system of
vernacular education.

www.insightsactivelearn.com 51
Total Marks : 200
Test-3 (Subject)
( INSTA Prelims Test Series 2021 )

1843-53: James Jonathan’s experiments in NorthWest Provinces (UP), as the lieutenant-


governor there, included opening one government school as model school in each tehsildari
and a normal school for teachers’ training for vernacular schools.

1853: In a famous minute, Lord Dalhousie expressed strong opinion in favour of


vernacular education. Hence Statement 1 is incorrect.

1854: Wood’s Despatch made the following provisions for vernacular education:

1. Improvement of standards

2. Supervision by government agency

3. Normal schools to train teachers

These gave impetus to the cause of vernacular education

1854-71: The government paid some attention to secondary and vernacular education. The
number of vernacular schools increased by more than five-fold.

1882: The Hunter Commission held that State should make special efforts for
extension and improvement of vernacular education. Mass education was to be seen as
instructing masses through vernaculars. Hence Statement 2 is correct.

1904: Education policy put special emphasis on vernacular education and increased grants for
it.

1929: Hartog Committee presented a gloomy picture of primary education.

1937: These schools received encouragement from Congress ministries.

63. Consider the following statements regarding the Paramahansa Mandali:


1. It was founded by Ramakrishna Paramahamsa.
2. It encouraged freedom of thought and rationality.

Which of the statements given above is/are correct?


A. 1 only
B. 2 only
C. Both 1 and 2
D. Neither 1 nor 2

Correct Answer : B

Answer Justification :

Paramahansa Mandali was a secret socio religious group, established in 1849, in

www.insightsactivelearn.com 52
Total Marks : 200
Test-3 (Subject)
( INSTA Prelims Test Series 2021 )

Bombay and is closely related to Manav Dharma Sabha which was found in 1844 in Surat. It
was started by Durgaram Mehtaji, Dadoba Pandurang and a group of his friends.
Hence, statement 1 is incorrect.

Besides believing that one god should be worshipped, the society also said real religion is
based on love and moral conduct. Freedom of thought was encouraged as was
rationality. Hence, statement 2 is correct.

The founders of the mandali were primarily interested in breaking caste rules. At
their meetings, food cooked by lower caste people was taken by the members. These
mandalis also advocated widow remarriage and women’s education. Branches of Paramahansa
Mandali existed in Poona, Satara and other towns of Maharashtra

64. Consider the following statements:


1. The Anglo-Nepalese war ended with the Treaty of Sagauli.
2. First Burma War ended with the Treaty of Yandabo.
3. Second-Anglo Afghan War ended with the Treaty of Gandamak.

Which of the statements given above is/are correct?


A. 1 and 2 only
B. 1 and 3 only
C. 2 and 3 only
D. 1, 2 and 3

Correct Answer : D

Answer Justification :

All the statements given above are correct.

Anglo-Nepalese Relations:

The Gorkhas wrested control of Nepal from the successors of Ranjit Malla of Bhatgaon in
1760. They began to expand their dominion beyond the mountains. They found it easier to
expand in the southern direction, as the north was well defended by the Chinese. In 1801, the
English annexed Gorakhpur which brought the Gorkhas’ boundary and the Company’s
boundary together. The conflict started due to the Gorkhas’ capture of Butwal and Sheoraj in
the period of Lord Hastings (1813-23). The war, ended in the Treaty of Sagauli, 1816 which
was in favour of the British.

First Burma War (1824-26):

The first war with Burma was fought when the Burmese expansion westwards and occupation
of Arakan and Manipur, and the threat to Assam and the Brahmaputra Valley led to continuous
friction along the ill-defined border between Bengal and Burma, in the opening decades of the

www.insightsactivelearn.com 53
Total Marks : 200
Test-3 (Subject)
( INSTA Prelims Test Series 2021 )

nineteenth century. The British expeditionary forces occupied Rangoon in May 1824 and
reached within 72 km of the capital at Ava. Peace was established in 1826 with the Treaty of
Yandabo.

Anglo-Afghan war:

The Treaty of Gandamak (May 1879) was signed with Yakub Khan, the eldest son of Sher Ali.
Treaty of Gandamak (May 1879). The treaty signed after the Second-Anglo Afghan War.

65. Which of the following countries is/are border Israel?


1. Lebanon
2. Syria
3. Egypt
4. Jordan

Select the correct answer using the code given below:


A. 1, 2 and 3 only
B. 2, 3 and 4 only
C. 1 and 4 only
D. 1, 2, 3 and 4

Correct Answer : D

Answer Justification :

All the countries given above border Israel.

According to the Green Line of the 1949 Armistice Agreements, Israel borders
Lebanon in the north, the Golan Heights and Syria in the northeast, the West Bank
and Jordan in the east, the Gaza Strip and Egypt in the southwest.

www.insightsactivelearn.com 54
Total Marks : 200
Test-3 (Subject)
( INSTA Prelims Test Series 2021 )

https://www.thehindu.com/opinion/editorial/limited-peace-the-hindu-editorial-on-uae-israel-pea
ce-agreement/article32359121.ece

66. Younghusband mission was sent to

A. Nepal
B. Tibet
C. Bhutan
D. Kashmir

Correct Answer : B

Answer Justification :

The British efforts to establish friendly and commercial relations with Tibet had not yielded
any result in the past and a deadlock had been reached by the time of Curzon’s arrival in India.
The Chinese suzerainty over Tibet was ineffective and Russian influence at Lhasa was
increasing. There were reports of Russian arms and ammunition coming into Tibet. Curzon
felt alarmed and sent a small Gorkha contingent under Colonel Younghusband on a
special mission to Tibet to oblige the Tibetans to come to an agreement. The Tibetans
refused to negotiate and offered non-violent resistance. Younghusband pushed his
way into Lhasa (August 1904) while the Dalai Lama fled.

Treaty of Lhasa (1904) Younghusband dictated terms to the Tibetan officials.

Hence, option (b) is correct.

67. Consider the following statements


1. Jagannath Shankar Seth was the active promoter of girls’ schools in Maharashtra.
2. Bhau Daji founded the Widow Remarriage Association in the 1850s.
3. Veerasalingam Pantulu promoted education in Madras region.

Which of the statements given above is/are correct?


A. 1 and 2 only
B. 3 only
C. 1 and 3 only
D. None

Correct Answer : C

Answer Justification :

www.insightsactivelearn.com 55
Total Marks : 200
Test-3 (Subject)
( INSTA Prelims Test Series 2021 )

Jagannath Shankar Seth and Bhau Daji were among the active promoters of girls’
schools in Maharashtra. Hence Statement 1 is correct.

Vishnu Shastri Pandit founded the Widow Remarriage Association in the 1850s.
Hence Statement 2 is incorrect.

Another prominent worker in this field was Karsondas Mulji who started the Satya Prakash in
Gujarati in 1852 to advocate widow remarriage.

Similar efforts were made by Professor D.K. Karve in western India and by
Veerasalingam Pantulu in Madras. Hence Statement 3 is correct.

Karve himself married a widow in 1893. He dedicated his life to the upliftment of Hindu
widows and became the secretary of the Widow Remarriage Association.

68. Consider the following statements regarding the Indian Councils Act of 1861:
1. It provided that the Viceroy shall nominate at least one Indian into his Executive Council.
2. It took way the legislative powers from the Bombay and Madras Presidencies.
3. It empowered the Viceroy to issue ordinances, but with the concurrence of the legislative
council, during an emergency.

Which of the statements given above is/are correct?


A. 1 and 2 only
B. 3 only
C. 1, 2 and 3
D. None

Correct Answer : D

Answer Justification :

None of the statements given above are correct.

The Indian Councils Act of 1861 is an important landmark in the constitutional and political
history of India.

Features of the Act of 1861:

1. It made a beginning of representative institutions by associating Indians with the


law-making process. It thus provided that the viceroy should nominate some Indians
as non-official members of his expanded council. In 1862, Lord Canning, the then
viceroy, nominated three Indians to his legislative council—the Raja of Benaras, the
Maharaja of Patiala and Sir Dinkar Rao.

2. It initiated the process of decentralisation by restoring the legislative powers to the


Bombay and Madras Presidencies. It thus reversed the centralising tendency that started
www.insightsactivelearn.com 56
Total Marks : 200
Test-3 (Subject)
( INSTA Prelims Test Series 2021 )

from the Regulating Act of 1773 and reached its climax under the Charter Act of 1833. This
policy of legislative devolution resulted in the grant of almost complete internal autonomy to
the provinces in 1937.

3. It also provided for the establishment of new legislative councils for Bengal, North-Western
Frontier Province (NWFP) and Punjab, which were established in 1862, 1866 and 1897
respectively.

4. It empowered the Viceroy to make rules and orders for the more convenient transaction of
business in the council. It also gave a recognition to the ‘portfolio’ system, introduced by Lord
Canning in 1859. Under this, a member of the Viceroy’s council was made in-charge of one or
more departments of the government and was authorised to issue final orders on behalf of the
council on matters of his department(s).

5. It empowered the Viceroy to issue ordinances, without the concurrence of the


legislative council, during an emergency. The life of such an ordinance was six
months

69. Who founded the women organization ‘Bharat Stree Mahamandal’?

A. Sarla Devi Chaudhurani


B. Pandita Ramabai Saraswati
C. Cornelia Sarabji
D. Tarabai Premchand

Correct Answer : A

Answer Justification :

Bharat Stree Mahamandal was a women's organisation in India founded by Sarala


Devi Chaudhurani in Allahabad in 1910. One of the primary goals of the organisation was
to promote female education, which at that time was not well developed. Other major
objectives included abolition of the purdah system and improvement in the socio-economic and
political status of woman all over India.

Hence, option (a) is correct.

70. Which among the following organizations/institute released the Envistats India 2020 report?

A. The Forest Survey of India (FSI)


B. The National Statistical Office (NSO)
C. The Central Statistics Office (CSO)
D. National Environment Protection Authority (NEPA)

www.insightsactivelearn.com 57
Total Marks : 200
Test-3 (Subject)
( INSTA Prelims Test Series 2021 )

Correct Answer : B

Answer Justification :

The National Statistical Office recently released the Envistats India 2020 report.
Hence, option (b) is correct.

Observations by the report:

After declining for two years, the average number of heat wave days increased 82.6
per cent year-on-year to 157 in 2019, with the highest number recorded in Rajasthan
(20 days) followed by Uttar Pradesh and Uttarakhand (13 days each), the state-wise
data in the EnviStats India 2020 report released by National Statistical Office (NSO) showed.

The data shows that acute respiratory infection led to 3,740 deaths in 2018 — the
highest in six years (according to these provisional estimates). The highest number of
deaths due to acute respiratory infection were reported in West Bengal (732), followed by
Uttar Pradesh (699), Andhra Pradesh (587), Delhi (492).

https://indianexpress.com/article/india/over-80-rise-in-heat-wave-days-in-2019-rajasthan-worst-
hit-6537261/

71. The founder of the independent principality of Awadh was

A. Nizam-ul-Mulk
B. Zulfikar Khan
C. Burhan-ul-Mulk
D. Murshid Kuli Khan

Correct Answer : C

Answer Justification :

Hyderabad:

The founder of the Asaf-Jah house of Hyderabad was Kilich Khan, popularly known as Nizam-
ul-Mulk. It was Zulfikar Khan who had first conceived the idea of an independent state in the
Deccan.

Awadh:

The founder of the independent principality of Awadh was Saadat Khan, popularly
known as Burhan-ul-Mulk. Saadat Khan was a Shia. He had joined in a conspiracy against
the Sayyid brothers, which resulted in his being given an increased mansab.

www.insightsactivelearn.com 58
Total Marks : 200
Test-3 (Subject)
( INSTA Prelims Test Series 2021 )

Hence, option (c) is correct.

Bengal:

Murshid Kuli Khan was the founder of the independent state of Bengal. He was a capable ruler
and made Bengal a prosperous state.

Kerala:

Martanda Varma established an independent state of Kerala with Travancore as his capital. He
extended the boundaries of his state from Kanyakumari to Cochin. He made efforts to organise
his army along the Western model and adopted various measures to develop his state.

72. Who was the first lady barrister of India?

A. Shaffi Tyabji
B. Cornelia Sarabji
C. Kamla Devi Chattopadhyaya
D. Lady Dorab Tata

Correct Answer : B

Answer Justification :

Cornelia Sorabji was an Indian woman who was the first female graduate from
Bombay University, the first woman to study law at Oxford University and the first
female advocate in India, and the first woman to practice law in India and Britain. In
2012, a bust of her was unveiled at Lincoln's Inn, London.

73. Consider the following statements regarding the Charter Act of 1853:

1. It separated the legislative and executive functions of the Governor-General’s council.


2. It ended the open competition system of selection and recruitment of civil servants.
3. It allowed East India Company (EIC) to retain the possession of Indian territories on trust for
the British Crown for next twenty years.

Which of the statements given above is/are correct?


A. 1 only
B. 2 and 3 only
C. 1 and 3 only
D. 2 only

www.insightsactivelearn.com 59
Total Marks : 200
Test-3 (Subject)
( INSTA Prelims Test Series 2021 )

Correct Answer : A

Answer Justification :

Charter Act of 1853:

This was the last of the series of Charter Acts passed by the British Parliament between 1793
and 1853. It was a significant constitutional landmark.

Features of the Act:

1. It separated, for the first time, the legislative and executive functions of the
Governor-General’s council. Hence, statement 1 is correct. It provided for addition of six
new members called legislative councillors to the council. In other words, it established a
separate Governor-General’s legislative council which came to be known as the Indian
(Central) Legislative Council. This legislative wing of the council functioned as a mini-
Parliament, adopting the same procedures as the British Parliament. Thus, legislation, for the
first time, was treated as a special function of the government, requiring special machinery
and special process.

2. It introduced an open competition system of selection and recruitment of civil


servants. The covenanted civil service was thus thrown open to the Indians also. Accordingly,
the Macaulay Committee (the Committee on the Indian Civil Service) was appointed in 1854.
Hence, statement 2 is incorrect.

3. It extended the Company’s rule and allowed it to retain the possession of Indian
territories on trust for the British Crown. But, it did not specify any particular period,
unlike the previous Charters. This was a clear indication that the Company’s rule could be
terminated at any time the Parliament liked. Hence, statement 3 is incorrect.

4. It introduced, for the first time, local representation in the Indian (Central)
Legislative Council. Of the six new legislative members of the governor general’s council,
four members were appointed by the local (provincial) governments of Madras, Bombay,
Bengal and Agra.

74. Consider the following statements


1. Nil Darpan was a Bengali play written by Dinabandhu Mitra in 1858–1859.
2. Raja Rammohan Roy was the author of Bahubivah book

Which of the statements given above is/are correct?


A. 1 only
B. 2 only
C. Both 1 and 2
D. Neither 1 nor 2

www.insightsactivelearn.com 60
Total Marks : 200
Test-3 (Subject)
( INSTA Prelims Test Series 2021 )

Correct Answer : A

Answer Justification :

Nil Darpan is a Bengali play written by Dinabandhu Mitra in 1858–1859. The play was
published in Dhaka in 1860, under a pseudonym of the author. The play was essential to
Nil Vidroha, better known as the Indigo Revolt of February–March 1859 in Bengal, when
farmers refused to sow indigo in their fields to protest against exploitative farming under the
British Raj.Hence Statement 1 is correct.

Ishwar Chandra Vidyasagar CIE, born Ishwar Chandra Bandyopadhyay, was an Indian
educator and social reformer. His efforts to simplify and modernise Bengali prose were
significant. He wrote the book 'Bahubivah'. Hence Statement 2 is incorrect.

75. Consider the following statements regarding the Island Development Agency (IDA):
1. It was constituted under the aegis of Union Ministry of Shipping in 2017 for the development
of islands.
2. Union Minister of Shipping is the ex-officio chairperson of the Island Development Agency
(IDA).

Which of the statements given above is/are correct?


A. 1 only
B. 2 only
C. Both 1 and 2
D. Neither 1 nor 2

Correct Answer : D

Answer Justification :

None of the statements given above is correct.

India has a total of 1,382 off-shore identified islands. The development potential of
these islands and maritime resources has been accorded high priority. The
Government has constituted Island Development Agency (IDA) an apex body, under
the chairmanship of Hon’ble Home Minister, in June 2017, which has mandated NITI
Aayog to steer the Holistic Development of Islands program.

https://www.pmindia.gov.in/en/news_updates/pm-launches-submarine-cable-connectivity-to-an
daman-nicobar-islands-cani/

www.insightsactivelearn.com 61
Total Marks : 200
Test-3 (Subject)
( INSTA Prelims Test Series 2021 )

https://www.niti.gov.in/niti/content/holistic-development-islands-islanders-benefits

76. Who among the following gave the title of Raja to Rammohan Rai?

A. Bahadur Shah I
B. Jahandar Shah
C. Shah Alam II
D. Akbar II

Correct Answer : D

Answer Justification :

Akbar II (1806-37):

Akbar II (22 April 1760 – 28 September 1837), also known as Akbar Shah II, was the
penultimate Mughal emperor of India. He reigned from 1806 to 1837. He was the
second son of Shah Alam II and the father of Bahadur Shah II.

Akbar had little de facto power due to the increasing British influence in India through the
East India Company. He sent Ram Mohan Roy as an ambassador to Britain and gave
him the title of Raja. During his regime, in 1835, the East India Company (EIC) discontinued
calling itself subject of the Mughal Emperor and issuing coins in his name. The Persian lines in
the company's coins to this effect were deleted.

Hence, option (d) is correct.

77. Consider the following events


1. First Anglo-Maratha War
2. Fourth Anglo-Mysore War
3. Second Sikh War

Which of the following is the correct chronological sequence of the above events?
A. 3-2-1
B. 3-1-2
C. 2-1-3
D. 1-2-3

Correct Answer : D

www.insightsactivelearn.com 62
Total Marks : 200
Test-3 (Subject)
( INSTA Prelims Test Series 2021 )

Answer Justification :

The First Anglo-Maratha War was the first of three Anglo-Maratha Wars wars fought
between the British East India Company and Maratha Empire in India. The war began with the
Treaty of Surat and ended with the Treaty of Salbai. (Period: 1775 – 1782)

The Fourth Anglo–Mysore War was a conflict in South India between the Kingdom of
Mysore against the British East India Company and the Hyderabad Deccan in 1798–99.

The Second Anglo-Sikh War (First War of Sikh Independence) was a military conflict
between the Sikh Empire and the British East India Company that took place in 1848
and 1849. It resulted in the fall of the Sikh Empire, and the annexation of the Punjab and
what subsequently became the North-West Frontier Province, by the East India Company.

78. Consider the following statements regarding the National Pharmaceutical Authority (NPA)
1. It is an independent body of experts under Ministry of Health and Family Welfare.
2. It implements and enforces the provisions of the Drugs Price Control Order.

Which of the statements given above is/are correct?


A. 1 only
B. 2 only
C. Both 1 and 2
D. Neither 1 nor 2

Correct Answer : B

Answer Justification :

National Pharmaceutical Pricing Authority (NPPA)

It was established in 1997 as an independent body of experts under Department of


Pharmaceuticals, Union Ministry of chemicals and Fertilizers. Hence Statement 1 is incorrect.

Functions are:

. To implement and enforce the provisions of the Drugs Price Control Order (DPCO),
1995/2013 in accordance with the powers delegated to it. Hence Statement 2 is correct.

. To undertake and/or sponsor relevant studies in respect of pricing of drugs/formulations.

. To monitor the availability of drugs, identify shortages if any, and to take remedial steps.

. To collect/maintain data on production, exports and imports, market share of individual


companies, profitability of companies etc. for hulk drugs and formulations.

. To deal with all legal matters arising out of the decisions of the Authority.

www.insightsactivelearn.com 63
Total Marks : 200
Test-3 (Subject)
( INSTA Prelims Test Series 2021 )

. To render advice to the Central Government on changes/revisions in the drug policy.

. To render assistance to the Central Government in the parliamentary matters relating to the
drug pricing.

79. Consider the following statements regarding provisions of the Charter Act of 1793
1. The Home Government members were to be paid out of British revenues.
2. The revenue administration was separated from the judiciary functions.

Which of the statements given above is/are correct?


A. 1 only
B. 2 only
C. Both 1 and 2
D. Neither 1 nor 2

Correct Answer : B

Answer Justification :

The Charter Act of 1793

● The Act renewed the Company’s commercial privileges for next 20 years.

● The Company, after paying the necessary expenses, interest, dividends, salaries, etc., from
the Indian revenues, was to pay 5 lakh pounds annually to the British government.

● The royal approval was mandated for the appointment of the governor-general, the
governors, and the commander-in- chief.

● Senior officials of the Company were debarred from leaving India without permission—doing
so was treated as resignation.

● The Company was empowered to give licences to individuals as well as the Company’s
employees to trade in India. The licences, known as ‘privilege’ or ‘country trade’, paved the
way for shipments of opium to China.

● The revenue administration was separated from the judiciary functions and this led
to disappearing of the Maal Adalats.

● The Home Government members were to be paid out of Indian revenues which
continued up to 1919. Hence, statement 1 is incorrect.

80. Recently, which of the following state reserved all government jobs to its state residents?

www.insightsactivelearn.com 64
Total Marks : 200
Test-3 (Subject)
( INSTA Prelims Test Series 2021 )

A. Maharashtra
B. Punjab
C. Madhya Pradesh
D. West Bengal

Correct Answer : C

Answer Justification :

The Madhya Pradesh government’s recent decision to reserve all government jobs for
“children of the state” raises questions relating to the fundamental right to equality.

While domicile-based reservations have been implemented in education, courts have been
reluctant to expand this to employment.

https://indianexpress.com/article/explained/domicile-based-job-quota-the-law-sc-rulings-and-sp
ecial-cases-6561814/

81. Consider the following statements regarding provisions of the Charter Act of 1813
1. The Company’s shareholders were given a 50 per cent dividend on the revenue of India.
2. The Company’s monopoly over trade in India ended.
3. Christian missionaries were permitted to come to India and preach their religion.

Which of the statements given above is/are correct?


A. 1 and 2 only
B. 1 and 3 only
C. 2 and 3 only
D. 1, 2 and 3

Correct Answer : C

Answer Justification :

The Charter Act of 1813:

In England, the business interests were pressing for an end to the Company’s monopoly over
trade in India because of a spirit of laissez-faire and the continental system by Napoleon by
which the European ports were closed for Britain. The 1813 Act sought to redress these
grievances—

● The Company’s monopoly over trade in India ended, but the Company retained the
trade with China and the trade in tea.

● The Company’s shareholders were given a 10.5 per cent dividend on the revenue of

www.insightsactivelearn.com 65
Total Marks : 200
Test-3 (Subject)
( INSTA Prelims Test Series 2021 )

India. Hence, statement 1 is incorrect.

● The Company was to retain the possession of territories and the revenue for 20 years more,
without prejudice to the sovereignty of the Crown. (Thus, the constitutional position of the
British territories in India was defined explicitly for the first time.)

● Powers of the Board of Control were further enlarged.

● A sum of one lakh rupees was to be set aside for the revival, promotion and encouragement
of literature, learning and science among the natives of India, every year. (This was an
important statement from the point of State’s responsibility for education.)

● The regulations made by the Councils of Madras, Bombay and Calcutta were now required
to be laid before the British Parliament. The constitutional position of the British territories in
India was thus explicitly defined for the first time.

● Separate accounts were to be kept regarding commercial transactions and territorial


revenues. The power of superintendence and direction of the Board of Control was not only
defined but also enlarged considerably.

● Christian missionaries were also permitted to come to India and preach their
religion.

82. With reference to the Carnatic Wars, consider the following statements
1. The First Carnatic war ended with signing of the treaty of Aix-la-Chapelle.
2. During second Carnatic war the French recalled Dupleix.

Which of the statements given above is/are correct?


A. 1 only
B. 2 only
C. Both 1 and 2
D. Neither 1 nor 2

Correct Answer : C

Answer Justification :

Both statements given above are correct.

The First Carnatic War had its background in the War of Austrian Succession (1740-48) which
was raging in Europe.

The First Carnatic war was ended with signing of the treaty of Aix-la-Chapelle. By the
terms of the treaty Madras was restored to the English. In this way the first phase of the
struggle between the English and the French on the Indian soil ended without any territorial
loss or gain to any of the Parties.

www.insightsactivelearn.com 66
Total Marks : 200
Test-3 (Subject)
( INSTA Prelims Test Series 2021 )

During second Carnatic war the French recalled Dupleix. Godehu was sent to
supersede Dupleix, and he arrived on August 1, 1754. Godehu at once opened
negotiations with the English and a treaty was signed stipulating that French or the English
would not interfere in the quarrels among the native princes, and both parties were left in
occupation of territories actually under their respective possession.

83. Which of the following is/are the functions of the National Testing Agency?
1. It aims to create a question bank for all subjects using the modern techniques.
2. It aims to establish a strong R&D culture as well as a pool of experts in different aspects of
testing.
3. It aims to undertake the reforms and training of school boards as well as other bodies where
the testing standards should be comparable with the entrance examinations.

Select the correct answer using the code given below


A. 1 only
B. 2 and 3 only
C. 1 and 2 only
D. 1, 2 and 3

Correct Answer : D

Answer Justification :

All the statements given above are correct.

National Testing Agency (NTA) has been established as a premier, specialist, autonomous and
self-sustained testing organization to conduct entrance examinations for admission/fellowship
in higher educational institutions.

FUNCTIONS

To identify partner institutions with adequate infrastructure from the existing schools
and higher education institutions which would facilitate conduct of online examinations
without adversely impacting their academic routine.
To create a question bank for all subjects using the modern techniques
To establish a strong R&D culture as well as a pool of experts in different
aspects of testing
To help individual colleges and universities in the field of testing and to provide training
and advisory services to the institutions in India. To provide quality testing services to
the academic institutions in India.
To develop a state of the art culture of testing in India by using domestic and
international expertise. To collaborate with international organizations like ETS to
achieve the same.

www.insightsactivelearn.com 67
Total Marks : 200
Test-3 (Subject)
( INSTA Prelims Test Series 2021 )

To undertake any other examination that is entrusted to it by the Ministries/Departments


of Government of India/State Governments.
To undertake the reforms and training of school boards as well as other bodies
where the testing standards should be comparable with the entrance
examinations.

https://www.nta.ac.in/functions

84. Consider the following statements regarding the Indian Civil Service Act, 1861:
1. This Act reserved certain offices for covenanted civil servants.
2. The examination was held in England in English language, based on classical learning of Greek
and Latin.
3. Satyendra Nath Tagore became the first Indian to qualify for the Indian Civil Service.

Which of the statements given above is/are correct?


A. 1 and 2 only
B. 1 and 3 only
C. 2 and 3 only
D. 1, 2 and 3

Correct Answer : D

Answer Justification :

Indian Civil Service Act, 1861:

This Act reserved certain offices for convenanted civil servants but the examination
was held in England in English language, based on classical learning of Greek and
Latin. The maximum permissible age was gradually reduced from 23 (in 1859) to 22 (in 1860)
to 21 (in 1866) and to 19 (1878). In 1863, Satyendra Nath Tagore became the first Indian
to qualify for the Indian Civil Service.

Statutory Civil Service:

In 1878-79, Lytton introduced the Statutory Civil Service consisting of one-sixth of


covenanted posts to be filled by Indians of high families through nominations by local
governments subject to approval by the secretary of State and the viceroy. But the system
failed and was abolished.

Government of India Act, 1935:

The 1935 Act recommended the establishment of a Federal Public Service Commission and
Provincial Public Service Commission under their spheres. But the positions of control and

www.insightsactivelearn.com 68
Total Marks : 200
Test-3 (Subject)
( INSTA Prelims Test Series 2021 )

authority remained in British hands and the process of Indianisation of the civil service did not
put effective political power in Indian hands since the Indian bureaucrats acted as the agents
of colonial rule.

85. Consider the following statements regarding the TRIFOOD Scheme


1. It is a joint initiative of Ministry of Food Processing Industry, Ministry of Tribal Affairs and
TRIFED.
2. Under this scheme, a tertiary value addition center will be setup in the rural areas.

Which of the statements given above is/are correct?


A. 1 only
B. 2 only
C. Both 1 and 2
D. Neither 1 nor 2

Correct Answer : C

Answer Justification :

Both the statements are correct.

TRIFOOD Scheme, a joint initiative of Ministry of Food Processing Industry, Ministry


of Tribal Affairs and TRIFED. Under this scheme a tertiary value addition center will
be set up in Jagdalpur in Chhattisgarh and Raigad in Maharashtra at a cost of
approximately Rs.11.00 crores. A highlight of this is the production of “Heritage
Mahua” drink.

The traditional Mahua tribal drink will be mainstreamed and marketed all over the Country
under this project. He praised the efforts of the Ministry of Tribal Affairs and TRIFED to take
all these programmes to the present level and hoped that TRIFED will leave no stone unturned
to reach the tribals residing in forest areas.

https://pib.gov.in/PressReleasePage.aspx?PRID=1566635

86. Consider the following statements regarding the First Carnatic War (1740-48)
1. Carnatic was the name given by the Arabs to the Coromandel Coast and its hinterland.
2. The First Carnatic War was an extension of the Anglo-French War in Europe.

Which of the statements given above is/are correct?


A. 1 only
B. 2 only
C. Both 1 and 2
D. Neither 1 nor 2

www.insightsactivelearn.com 69
Total Marks : 200
Test-3 (Subject)
( INSTA Prelims Test Series 2021 )

Correct Answer : B

Answer Justification :

First Carnatic War (1740-48)

Carnatic was the name given by the Europeans to the Coromandel Coast and its
hinterland. Hence statement 1 is incorrect.

The First Carnatic War was an extension of the Anglo-French War in Europe which
was caused by the Austrian War of Succession. Hence statement 2 is correct.

Immediate Cause Although France, conscious of its relatively weaker position in India, did not
favour an extension of hostilities to India, the English navy under Barnet seized some French
ships to provoke France. France retaliated by seizing Madras in 1746 with the help of the fleet
from Mauritius, the Isle of France, under Admiral La Bourdonnais, the French governor of
Mauritius. Thus began the first Carnatic War.

Result The First Carnatic War ended in 1748 when the Treaty of Aix-La Chapelle was signed
bringing the Austrian War of Succession to a conclusion.

Under the terms of this treaty, Madras was handed back to the English, and the French, in
turn, got their territories in North America.

87. Consider the following statements regarding the Policy of Ring-Fence


1. Lord Wellesley followed a policy of ring-fence.
2. It aimed at creating buffer zones to defend the Company’s frontiers.
3. Policy of subsidiary alliance was an extension of the ring-fence system.

Which of the statements given above is/are correct?


A. 1 only
B. 2 and 3 only
C. 1 and 3 only
D. None

Correct Answer : B

Answer Justification :

The Policy of Ring-Fence:

Warren Hastings took charge as the governor-general at a critical period of British rule when
the British were to encounter the powerful combination of the Marathas, Mysore and

www.insightsactivelearn.com 70
Total Marks : 200
Test-3 (Subject)
( INSTA Prelims Test Series 2021 )

Hyderabad. He followed a policy of ring-fence which aimed at creating buffer zones to


defend the Company’s frontiers. Hence Statement 1 is incorrect.

Broadly speaking, it was the policy of defence of their neighbors’ frontiers for
safeguarding their own territories. Hence Statement 2 is correct.

This policy of Warren Hastings was reflected in his war against the Marathas and Mysore.

Wellesley’s policy of subsidiary alliance was, in fact, an extension of the ring-fence


system which sought to reduce the Indian states into a position of dependence on the
British government. Hence Statement 3 is correct.

88. Identify the national park based on the given facts


1. It is recognized as an Important Bird Area by BirdLife International.
2. The park was declared as a World Heritage Site by UNESCO in 1985.
3. The sanctuary hosts two-thirds of the world’s great one-horned rhinoceroses.

Select the correct answer using the code given below:


A. Kaziranga National Park
B. Manas National Park
C. Dibru Saikhowa National Parrk
D. Nameri National Park

Correct Answer : A

Answer Justification :

Kaziranga National Park is a national park in the Golaghat, Karbi Anglong and Nagaon
districts of the state of Assam, India. The sanctuary, hosts two-thirds of the world's great
one-horned rhinoceroses.

In the year 1985, the park was declared as a World Heritage Site by UNESCO.

It is recognized as an Important Bird Area by Birdlife International.

It was declared as Tiger Reserve in 2006. The sanctuary hosts two-thirds of the world’s great

www.insightsactivelearn.com 71
Total Marks : 200
Test-3 (Subject)
( INSTA Prelims Test Series 2021 )

one-horned rhinoceroses.

89. Consider the following statements regarding the Treaty of Allahabad


1. Nawab Shuja-ud-Daula agreed to surrender Allahabad and Kara to Emperor Shah Alam II.
2. Shah Alam II agreed to issue a farman granting the diwani of Bengal, Bihar and Orissa to the
East India Company.

Which of the statements given above is/are correct?


A. 1 only
B. 2 only
C. Both 1 and 2
D. Neither 1 nor 2

Correct Answer : C

Answer Justification :

Both the statements are correct.

The Treaty of Allahabad:

Robert Clive concluded two important treaties at Allahabad in August 1765—one with the
Nawab of Awadh and the other with the Mughal Emperor, Shah Alam II.

Nawab Shuja-ud-Daula agreed to:


(i) surrender Allahabad and Kara to Emperor Shah Alam II;
(ii) pay Rs 50 lakh to the Company as war indemnity; and
(iii) give Balwant Singh, Zamindar of Banaras, full possession of his estate.

Shah Alam II agreed to:


(i) reside at Allahabad, to be ceded to him by the Nawab of Awadh, under the
Company’s protection;
(ii) issue a farman granting the diwani of Bengal, Bihar and Orissa to the East India Company
in lieu of an annual payment of Rs 26 lakh; and
(iii) a provision of Rs 53 lakh to the Company in return for nizamat functions (military defence,
police, and administration of justice) of the said provinces

90. What is the correct sequence of island located in Indian Ocean as one proceeds from Lakshadweep?

A. Maldives-Chagos Archipelago-Seychelles-Mauritius
B. Maldives-Mauritius- Seychelles-Chagos Archipelago

www.insightsactivelearn.com 72
Total Marks : 200
Test-3 (Subject)
( INSTA Prelims Test Series 2021 )

C. Mauritius-Maldives- Seychelles-Chagos Archipelago


D. Mauritius- Chagos Archipelago-Seychelles-Maldives

Correct Answer : A

Answer Justification :

91. Consider the following statements


1. Haidar Ali died of cancer during first Anglo-Mysore war.
2. Lord Ripon took control of Mysore on grounds of misgovernance in 1831.

Which of the statements given above is/are correct?


A. 1 only
B. 2 only
C. Both 1 and 2
D. Neither 1 nor 2

Correct Answer : D

Answer Justification :

Both the statements are incorrect.

Haidar Ali died of cancer on December 7, 1782 during second Anglo-Mysore war. Tipu
Sultan, carried on the war for one year without any positive outcome. Fed up with an

www.insightsactivelearn.com 73
Total Marks : 200
Test-3 (Subject)
( INSTA Prelims Test Series 2021 )

inconclusive war, both sides opted for peace, negotiating the Treaty of Mangalore (March,
1784) under which each party gave back the territories it had taken from the other

Mysore after Tipu:

● Wellesley offered Soonda and Harponelly districts of Mysore Kingdom to the Marathas,
which the latter refused.

● The Nizam was given the districts of Gooty and Gurramkonda.

● The English took possession of Kanara, Wynad, Coimbatore, Dwaraporam and


Seringapatam.

● The new state of Mysore was handed over to the old Hindu dynasty (Wodeyars) under a
minor ruler Krishnaraja III, who accepted the subsidiary alliance.

● In 1831 William Bentinck took control of Mysore on grounds of misgovernance.

● In 1881 Lord Ripon restored the kingdom to its ruler

92. Who among the following freedom fighters wrote the book regarding the Revolt of 1857 as ‘The War of Indian
Independence’?

A. Bal Gangadhar Tilak


B. Surendranth Banerjea
C. V.D. Savarkar
D. Subash Chandra Bose

Correct Answer : C

Answer Justification :

The Indian War of Independence is an Indian nationalist history of the 1857 revolt by
Vinayak Damodar Savarkar that was first published in 1909.

V.D. Savarkar, in his book ‘The War of Indian Independence’, published in 1909, argued that
what the British had till then described as merely mutiny was, in fact, a war of independence,
much like the American War of independence. Despite the fact that the English-educated
middle class played no role in the rebellion, nationalist historians championed this argument
as the First War of Indian Independence.

Hence, option (c) is correct.

www.insightsactivelearn.com 74
Total Marks : 200
Test-3 (Subject)
( INSTA Prelims Test Series 2021 )

93. Dehing Pataki Wildlife Sanctuary, sometimes seen in the news, is located in

A. Arunachal Pradesh
B. Assam
C. West Bengal
D. None of the above

Correct Answer : B

Answer Justification :

Dehing Patkai Wildlife Sanctuary is located in the Dibrugarh and Tinsukia Districts of
Assam and covers an area of 111.19 km² rainforest. It was declared a sanctuary on 13 June
2004. It is located in the Dehing patkai landscape which is a dipterocarp-dominated lowland
rainforest.

https://www.telegraphindia.com/north-east/aasu-protests-over-coal-mining-in-tinsukia/cid/1774
817

94. The Lottery Committee (1817) was concerned with:

A. Relationship with Princely States


B. Civil Service
C. Town planning
D. Gambling

Correct Answer : C

Answer Justification :

The Lottery Committee was set up in 1817 after the departure of Lord Wellesley. It
carried on the work of town planning with help of the government. It was named as the
Lottery committee because it raised funds through public lotteries. However, it used these
funds for the improvement of the town.

Hence, option (c) is correct.

95. With reference to the Global Geo-Park, Consider the following statements
1. They are single, unified geographical areas where sites and landscapes of international
geological significance are managed with a holistic concept of protection, education and
sustainable development.

www.insightsactivelearn.com 75
Total Marks : 200
Test-3 (Subject)
( INSTA Prelims Test Series 2021 )

2. It empowers local communities and gives them the opportunity to develop cohesive
partnerships with the common goal of promoting the area’s significant geological processes,
features etc.

Which of the statements given above is/are correct?


A. 1 only
B. 2 only
C. Both 1 and 2
D. Neither 1 nor 2

Correct Answer : C

Answer Justification :

Both the statements are correct.

UNESCO Global Geoparks are single, unified geographical areas where sites and
landscapes of international geological significance are managed with a holistic
concept of protection, education and sustainable development.

A UNESCO Global Geopark uses its geological heritage, in connection with all other aspects of
the area’s natural and cultural heritage, to enhance awareness and understanding of key
issues facing society, such as using our earth’s resources sustainably, mitigating the effects of
climate change and reducing natural disasters-related risks.

UNESCO Global Geoparks empower local communities and give them the opportunity
to develop cohesive partnerships with the common goal of promoting the area’s
significant geological processes, features, periods of time, historical themes linked to
geology, or outstanding geological beauty.

http://www.unesco.org/new/en/natural-sciences/environment/earth-sciences/unesco-global-geo
parks/frequently-asked-questions/what-is-a-unesco-global-geopark/

96. 'No religion, No caste and No God for mankind’ is a statement ascribed to:

A. E. V. Ramaswamy Naicker
B. Sahadran Ayyappan
C. Shri Narayana Guru
D. Mahatma Gandhi

Correct Answer : B

Answer Justification :

www.insightsactivelearn.com 76
Total Marks : 200
Test-3 (Subject)
( INSTA Prelims Test Series 2021 )

Sahodaran Ayyappan, was a social reformer, thinker, rationalist, journalist, and


politician from Kerala, India.

A vocal follower of Sree Narayana Guru, he was associated with a number of events related to
the Kerala reformation movement and was the organizer of Misra Bojana in Cherai in 1917.
He founded Sahodara Sangam, and the journal Sahodaran and was the founder editor
of the magazine Yukthivadhi.

Guru Sri Narayana Guru preached the doctrine of ' One caste, One religion, One God’. His
disciple Ayyapan, changed into 'no religion, no caste and no God for mankind.

Hence, option (b) is correct.

97. Consider the following statements regarding the Islamic Reform Movements during 19th century in British
India:
1. Ahmadiya movement had progressive outlook which opposed polygamy, veiling of women and
classical rules of divorce.
2. Deoband Movement aimed at propagating the pure teachings of the Quran and Hadis among
Muslims.

Which of the statements given above is/are not correct?


A. 1 only
B. 2 only
C. Both 1 and 2
D. Neither 1 nor 2

Correct Answer : A

Answer Justification :

The Ahmadiya movement founded by Mirza Ghulam Ahmed (1835–1908) in 1889. While
emphasizing the return to the original principles enunciated in the Quran, Ghulam Ahmed
became controversial when he claimed to be a Messiah, which was considered heretical by
mainstream Islam. But he won many converts. His primary work was to defend Islam against
the polemics of the Arya Samaj and the Christian missionaries. In social morals the
Ahmadiya movement was conservative, adhering to polygamy, veiling of women, and
the classical rules of divorce. Hence, statement 1 is incorrect.

The Deoband movement was organised by the orthodox section among the Muslim
ulemas as a revivalist movement with the twin objective of propagating the pure
teachings of the Quran and Hadis among Muslims. The movement was established in
Deoband in Saranpur district (by Mohammad Qasim Nanotavi (1833-1877) and Rashid Ahmed
Gangohi (1828–1905) to train religious leaders for the Muslim community. Hence, statement
2 is correct.

www.insightsactivelearn.com 77
Total Marks : 200
Test-3 (Subject)
( INSTA Prelims Test Series 2021 )

98. Consider the following statements regarding the International Thermonuclear Experimental Reactor (ITER)
Project
1. It is an experimental fusion facility construction in Europe.
2. India is not a part of this project.
3. ITER will be the first fusion device to test the integrated technologies, materials and physics
regimes necessary for the commercial production of fusion-based electricity.

Which of the statements given above is/are correct?


A. 1 and 2 only
B. 3 only
C. 1 and 3 only
D. 1, 2 and 3

Correct Answer : C

Answer Justification :

ITER is an experimental fusion reactor facility under construction in Cadarache,


South of France to prove the feasibility of nuclear fusion for future source of energy. Hence
Statement 1 is correct.

ITER partners are the European Union (EU), China, India, Japan, South Korea, Russia
and the United States of America. European Union being the host party contributes
45% while the rest of the parties contribute 9% each. Hence Statement 2 is incorrect.

ITER will be the first fusion device to produce net energy. ITER will be the first fusion
device to maintain fusion for long periods of time. And ITER will be the first fusion device
to test the integrated technologies, materials, and physics regimes necessary for the
commercial production of fusion-based electricity. Hence Statement 3 is correct.

Most of these contributions are through ‘in-kind’ procurement of ¡TER components. India
formally joined the ITER Project in 2005 and the ITER Agreement between the partners was
signed in 2006.

ITER Organization (10) is the central team responsible for construction at site and operation,
while the ITER partners created their own domestic agencies to deliver their commitments to
ITER. ITER-India is the Indian domestic agency.

www.insightsactivelearn.com 78
Total Marks : 200
Test-3 (Subject)
( INSTA Prelims Test Series 2021 )

https://www.iter.org/proj/inafewlines

99. Consider the following pairs of centres of


Revolt of 1857 and leaders associated:
Revolt Centre: Leaders:
1. Bihar Begum Hazrat Mahal
2. Lucknow Nana Sahib
3. Jhansi Laxmibai

Which of the pairs given above is/are correctly matched?


A. 1 and 2 only
B. 3 only
C. 1 and 3 only
D. 2 and 3 only

Correct Answer : B

Answer Justification :

The rebellion 1857 first began as a mutiny in Barrackpore (near Calcutta). Mangal Pandey
murdered his officer in January 1857 and a mutiny broke out there. In the following month, at
Meerut, of the 90 sepoys who were to receive their cartridges only five obeyed orders. On 10
May three sepoy regiments revolted, killed their officers, and released those who had been
imprisoned. The next day they reached Delhi, murdered Europeans, and seized that city. The
rebels proclaimed Bahadur Shah II as emperor

Centres of Revolt and Leaders:

Delhi - General Bakht Khan

Kanpur - Nana Saheb

Lucknow - Begum Hazrat Mahal (Hence pair 2 is incorrectly matched)

Bareilly - Khan Bahadur

Bihar - Kunwar Singh (Hence, pair 1 is incorrectly matched)

Faizabad - Maulvi Ahmadullah

Jhansi - Rani Laxmibai (Hence, pair 3 is correctly matched)

Baghpat - Shah Mal

www.insightsactivelearn.com 79
Total Marks : 200
Test-3 (Subject)
( INSTA Prelims Test Series 2021 )

100. Genetic Engineering Appraisal Committee (GEAC) is constituted under the aegis of

A. The Forest (Conservation) Act, 1980


B. The Wildlife Protection Act, 1972
C. The Water (Prevention and Control of pollution) Act, 1974
D. The Environment (Protection) Act, 1986

Correct Answer : D

Answer Justification :

The Genetic Engineering Appraisal Committee (GEAC) functions in the Ministry of


Environment, Forest and Climate Change (MoEF&CC). As per Rules, 1989, it is responsible for
appraisal of activities involving large scale use of hazardous microorganisms and recombinants
in research and industrial production from the environmental angle.

Genetic Engineering Appraisal Committee (GEAC) is constituted under the aegis of


The Environment (Protection) Act, 1986

The committee is also responsible for appraisal of proposals relating to release of genetically
engineered (GE) organisms and products into the environment including experimental field
trials.

GEAC is chaired by the Special Secretary/Additional Secretary of MoEF&CC and co-chaired by


a representative from the Department of Biotechnology (DBT). Presently, it has 24 members
and meets every month to review the applications in the areas indicated above.

https://geacindia.gov.in/about-geac-india.aspx

www.insightsactivelearn.com 80

You might also like